MBA MFT Exam Study Guide

Ace your homework & exams now with Quizwiz!

You've developed a linear demand model for your product showing that QD = 50 - 4P + 2M, where P is the good's price and M is income. If M = 25 and P = 15, what is own price elasticity of demand? A. -1.5 B. -4 C. -0.5 D. -2.25

A. -1.5

You've been running regressions to estimate demand for your product. You're looking over the coefficients - which of the following should give you cause for concern? A. A positive coefficient on your good's own price. B. A positive coefficient on your rival's price. C. A positive coefficient on income. D. A positive coefficient on advertising.

A. A positive coefficient on your good's own price.

Now this time, suppose sport utility vehicles and small pick-up trucks are substitutes in both production and consumption. An increase in the price of sport utility vehicles will result in... A. A rise in the price of small pick-up trucks, and an ambiguous change in the equilibrium quantity of the trucks. B. A fall in the price of small pick-up trucks, and an ambiguous change in the equilibrium quantity of the trucks. C. A rise in the equilibrium quantity of small pick-up trucks, and an ambiguous change in the price of the trucks. D. A fall in the equilibrium quantity of small pick-up trucks, and an ambiguous change in the price of the trucks.

A. A rise in the price of small pick-up trucks, and an ambiguous change in the equilibrium quantity of the trucks.

Which of the following is certain to fall as output rises? A. Average fixed cost B. Average variable cost

A. Average fixed cost

Your company sells annual subscriptions to an online movie service for $100, and you decide to lower your price by $12 to boost sales. Rivals do not change their prices, and your sales go from 600,000 to 800,000. What can we conclude? A. Demand for your service is elastic, and the decrease in price increased your total revenue. B. Demand for your service is inelastic, and the decrease in price increased your total revenue. C. Demand for your service is elastic, and the decrease in price decreased your total revenue. D. Demand for your service is inelastic, and the decrease in price decreased your total revenue.

A. Demand for your service is elastic, and the decrease in price increased your total revenue.

What do "isoquants" show? A. Different combinations of inputs that can be used to produce a certain quantity of output. B. The relationship between output on one axis and an input like labor on the other. C. How different levels of production affect total cost.

A. Different combinations of inputs that can be used to produce a certain quantity of output.

Again, suppose Olivia leaves her old job, and starts up a new business as proprietor of a small bakery. She works long hours alongside her hired help, whom she pays a good hourly wage. She also incurs the costs of a renting her facilities and equipment, and of course the goods used in baking; after her expenses are paid, she takes home whatever is left, with some months being better than others - and it works out to about $4000 on average. Based on this information, to find an approximation of her monthly economic profit, you would need to know... A. How much she earned at her old job. B. How much she pays for her current business costs. C. How much she earned at her old job AND how much she pays for her current business costs. D. How much gross revenue her business generates each month.

A. How much she earned at her old job.

If a firm's production function is Q = 100*L - L2, where Q is units of output and L is daily units of labor input, then its marginal product of labor is MPL = 100 - 2*L. If the firm sells its output in a competitive market for $5 each, what is the value of the marginal product when L = 25? A. More than $100. B. $100 or less.

A. More than $100.

Which of the following is an inverse demand function? A. P = 100 - 2QD B. QD = 100 - 2P C. QD = 50/P D. QD = 100 - 2P + 5M + 10 A.5

A. P = 100 - 2QD

From the text questions at the end of chapter 1: Southwest Airlines begins a "Bags Fly Free" campaign, charging no fees for the first and second checked bags. What kind of competition does this situation best represent? A. Producer - producer rivalry B. Consumer - consumer rivalry C. Consumer - producer rivalry D. None of these

A. Producer - producer rivalry

A coffee roaster sells two kinds of coffee - its "regular" brand and its "gourmet" variety. The regional demand for the regular coffee was found to be QD = 5000 - 20 PR - 3 M + 5 PG where PR is the price of regular coffee, M is income in the region, and PG is the price of gourmet coffee. What can we determine from this information? A. Regular coffee is an inferior good, and is a substitute for gourmet coffee. B. Regular coffee is a normal good, and is a substitute for gourmet coffee. C. Regular coffee is an inferior good, and is a complement for gourmet coffee. D. Regular coffee is a normal good, and is a complement for gourmet coffee.

A. Regular coffee is an inferior good, and is a substitute for gourmet coffee.

The board of directors for a public company has decided that the company will prioritize providing $10 million to the local community as part of its corporate social responsibility program, but that its overall aim is to maximize shareholder value through maximized profits. In terms of goals and constraints, the best way to formalize the board's objective(s) is to say... A. The company seeks to maximize shareholder value, subject to a constraint of providing $10 million to the community. B. The company seeks to maximize its contributions to the community, subject to the constraint of providing adequate financial value to its shareholders. C. The company seeks to maximize both its profits and its contributions to the community, subject to normal business operations. D. The company seeks to operate normally, subject to the constraints of maximizing shareholder value and making a $10 million contribution to the community.

A. The company seeks to maximize shareholder value, subject to a constraint of providing $10 million to the community.

Look carefully at formula describing the present value of a firm as a function of the firm's current profits, the market interest rate, and the firm's growth rate: This assumes i > g, as noted in the text. What happens as g gets bigger and becomes almost as big as i? A. The firms present value explodes towards infinite value! B. The firm's value collapses towards zero!

A. The firms present value explodes towards infinite value!

In a short run production function, one worker can make 12 widgets, and two workers can make 20 widgets. What can we conclude? A. The marginal product of labor is declining. B. The marginal product of labor is greater than the average product of labor for two workers.

A. The marginal product of labor is declining.

Two regressions have the same R2. The first uses 200 observations and 10 estimated coefficients; the second has 200 observations and 20 estimated coefficients. Which one has a higher adjusted R2? A. The one with 10 estimated coefficients. B. The one with 20 estimated coefficients. C. Both will have the same adjust R2 because both have the same number of observations. D. We can't tell because we don't know the F statistic.

A. The one with 10 estimated coefficients.

Cross price elasticity between your product and another firm's product is found to be -0.8. The other firm lowers the price of its product. Which of the following is correct, concerning a profit maximizing optimal response by you, holding other factors constant? A. The products are complements, and you should raise your price. B. The products are complements, and you should lower your price. C. The products are substitutes, and you should raise your price. D. The products are substitutes, and you should lower your price.

A. The products are complements, and you should raise your price.

In a competitive market, do workers get paid the value of their marginal product or the value of their average product? A. Value of the marginal product B. Value of the average product

A. Value of the marginal product

Under what circumstances will the "full economic price" of a good be greater than the "dollar price" of a good? A. When a shortage due to a price ceiling causes people to wait in line for goods with limited availability. B. When a shortage due to a price floor causes sellers to discount their products to move inventory. C. When a surplus due to a price ceiling causes people to wait in line for goods with limited availability. D. When a surplus due to a price floor causes sellers to discount their products to move inventory.

A. When a shortage due to a price ceiling causes people to wait in line for goods with limited availability.

A market that was previously in equilibrium experiences a change due to an increase in demand, resulting in higher market prices. Sketching these change in a demand and supply diagram to show the new price/quantity equilibrium is an exercise in... A. comparative statics. B. welfare analysis. C. marginal analytics. D. regression estimation.

A. comparative statics.

Accounting profits are likely to be ________________ than economic profits because economic costs include __________________ costs that are ignored in accounting profits. A. greater, implicit B. greater, explicit C. smaller, implicit D. smaller, explicit

A. greater, implicit

Rivalry tends to be __________________ intense (and hence the likelihood of sustaining profits is ________________) in concentrated industries - that is, those with relatively few firms. A. less, higher B. more, higher C. less, lower D. more, lower

A. less, higher

The text notes that "complementarities also affect industry profitability." The authors mean this in a... A. positive way - the more complements to an industry's output, the higher industry profit will be. B. negative way - the more complements to an industry's output, the lower industry profit will be.

A. positive way - the more complements to an industry's output, the higher industry profit will be.

Picture an inverse demand function on a graph (with P on the vertical axis, and Q on the horizontal axis, as usual). Suppose the function is a downward-sloping straight line. At the upper left, where the demand function is close to intersecting with the price axis, own price elasticity is... A. relatively big in absolute value, and demand is elastic. B. relatively big in absolute value, and demand is inelastic. C. relatively small in absolute value, and demand is elastic. D. relatively small in absolute value, and demand is inelastic.

A. relatively big in absolute value, and demand is elastic.

If your short run production function features diminishing marginal product for your variable input, then your marginal cost of production will... A. rise as output increases. B. fall as output increases.

A. rise as output increases.

If the marginal product of the last worker hired is greater than the average product over all workers, then average product must be... A. rising. B. falling.

A. rising.

A lawn care and landscaping service uses workers and lawn mowers to conduct its business. The company can readily add workers or cut hours as needed for flexibility, but cannot alter its fleet of mowers during the season, as these require special orders, delivery time, and approval from a parent company. Production decisions for this firm this summer are best thought of as... A. short run decisions. B. long run decisions.

A. short run decisions.

As shown in the text, cost functions show total cost as a function of... A. the quantity of output from the production process. B. the quantities of inputs in the production process.

A. the quantity of output from the production process.

You plan to subscribe to the Economist Magazine for the next three years. The annual subscription is $100 per year, and not expected to change, with the first payment due now, the second a year from now, and the third two years from now. They also offer a two year subscription for $175, and a three year subscription for $250. Because you do all of your banking with pawn shops and use high interest credit cards, the relevant interest rate for you is 25%. Which subscription combination offers you the best (present) value? A. Buy the three year subscription now. B. Buy the two year subscription now, and then buy a separate one year subscription two years from now. C. Pay for the one year subscription each year as you go. D. They all work out to be the same value, so it doesn't matter which one you choose.

B. Buy the two year subscription now, and then buy a separate one year subscription two years from now.

Suppose demand and supply conditions in a market are given by P = 12 - 2QD and P = 3 + QS respectively. In equilibrium, a market will usually generate both consumer and producer surplus. Who gets the most surplus in this market? A. Consumers and producers split the surplus equally. B. Consumers get more surplus than producers. C. Producers get more surplus than consumers. D. There is neither consumer nor producer surplus in this market.

B. Consumers get more surplus than producers.

Which combination of the following both tend to make a good's own price elasticity of demand more elastic? A. Having a longer time horizon for purchasing decisions, and being a smaller share of the household budget. B. Having many substitutes available, and being a larger share of the household budget. C. Having few substitutes available, and having a shorter time horizon for purchasing decisions. D. Having a shorter time horizon for purchasing decisions, and being a larger share of the household budget.

B. Having many substitutes available, and being a larger share of the household budget.

A firm is making a long run decision about getting the optimal mix of labor and capital. Suppose the marginal product of capital is 50, the marginal product of labor is 20, the cost of capital (r) is 20, and the cost of labor (w) is 10. What should the firm do? A. Hire more labor and release some capital. B. Hire more capital and release some labor.

B. Hire more capital and release some labor.

The text introduces a production function of the form, Q = aK + bL. This production function indicates... A. K and L are perfect complements and must be used in proportion to the values of a and b. B. K and L are perfect substitutes and output can be generated with either K or L.

B. K and L are perfect substitutes and output can be generated with either K or L.

A firm's total product function is Q = 100*L - L2 for 0 < L < 50, where L is the number of daily workers. This also means that the firm's marginal product of labor is MPL = 100 - 2*L for that same range of labor. If the daily wage rate is $100 and the price of output is $5 per unit of Q, then how many workers should the firm hire? Pick the right range in which the answer lies. A. Twenty five workers or fewer. B. More than 25, but fewer than 50.

B. More than 25, but fewer than 50.

A service firm pays its workers $10 per hour, which is the going wage rate in a competitive labor market. The firm sells its service for $75 per unit and has all the raw materials and has plenty of capacity to produce its service, in terms of machines and other equipment. It will take a worker 8 hours to produce an additional unit of service. Should the firm hire another worker? A. Yes B. No

B. No

If a production process requires labor and capital, and you add the average product of labor to the average product of capital, do you get "total product"? A. Yes B. No

B. No

Other things remaining the same, does the present value of a stream of income increase or decrease with an increase in the interest rate? A. The PV increases. B. The PV decreases. C. We cannot tell without more information.

B. The PV decreases.

Consider the demand for "pet rocks" (you can look these up on the Internet - they were a thing back in the 1970s). The demand for pet rocks is given by QD = 100 - 20 P + 50 A where Q is the quantity, P is the price, and A is advertising. Suppliers in the industry raise the price from $3 to $4, and raise advertising from 1 unit to 2 units. An industry newsletter says: "Prices are up; demand is up! Pet rocks break through the law of demand! Economists are stymied!" What's going on? A. The pet rock market is one of the rare occasions when the law of demand is actually violated. B. The pet rock market does not violate the law of demand - raising the price does not lead to an increase in demand. C. Economists are easily stymied, so this is not a surprise. (You should not pick this answer, even if you think it is true!)

B. The pet rock market does not violate the law of demand - raising the price does not lead to an increase in demand.

Again, suppose sport utility vehicles and small pick-up trucks are substitutes in production. Other things equal, what effect would an increase in the price of sport utility vehicles have on the price of small pick-up trucks? A. No effect. B. The price of small pick-up trucks should rise. C. The price of small pick-up trucks should fall. D. The price of small pick-up trucks should change, but we cannot tell in which direction.

B. The price of small pick-up trucks should rise.

A researcher is comparing two regressions to describe a product's demand function. The Excel spreadsheet for the first regression shows an F statistic of 12.33 and a "Significance F" of 0.009825. The second regression shows an F statistic of 106.29 and a "Significance F" of 0.000067. Which one shows a higher level of statistical significance and thus better overall "fit"" A. The first regression. B. The second regression.

B. The second regression.

You've run a regression to find the demand function for your product, and it includes a rival's price as one of the determining factors. Which of the following is most strongly supportive of dropping the rival's price out of the equation, because it may not be an important factor in demand for your product? A. Your regression has an R2 of 0.5. B. The t-stat on the rival's price is 0.8. C. The coefficient on the rival's price is 1.5. D. The p-value on the rival's price is 0.05.

B. The t-stat on the rival's price is 0.8.

Under what circumstances would a surplus exist in a competitive market? A. There is a legal price floor below the equilibrium price. B. There is a legal price floor above the equilibrium price. C. There is a legal price ceiling below the equilibrium. D. There is a legal price ceiling above the equilibrium.

B. There is a legal price floor above the equilibrium price.

One of the principles identified in the text claims that profits are... A. a reward for meeting the needs and wants of society. B. a signal to indicate society's most valued use of resources. C. the price society pays to get business people to work for them. D. the symbol of greed that ultimately spells doom for capitalism.

B. a signal to indicate society's most valued use of resources.

Chapter 3, "Quantitative Demand Analysis," is primarily about... A. describing the relationship between accounting theory - the numbers side of business - and management practice, i.e., the qualitative component of running a company. B. calculating and interpreting elasticities and using statistical regression analysis to understand the demand for products and services. C. learning formulas for the calculation of profits, total revenue, net revenue, earnings before interest and taxes, and earning before interest, taxes, depreciation, and amortization. D. computing the national income accounts, including gross domestic product, gross national product, net national product, national income, and all of the various components of national income.

B. calculating and interpreting elasticities and using statistical regression analysis to understand the demand for products and services.

Suppose you take a six month unpaid leave of absence from your well-paying job to finish your graduate degree. You pay $10,000 of tuition during that time. The opportunity cost of this action is best thought of as... A. $10,000. B. considerably more than $10,000. C. much less than $10,000.

B. considerably more than $10,000.

You lower your price 2% and get a 4% increase in the quantity demanded of your product. This indicates that demand is... A. elastic and marginal revenue is negative. B. elastic and marginal revenue is positive. C. inelastic and marginal revenue is negative. D. inelastic and marginal revenue is positive.

B. elastic and marginal revenue is positive.

Your professor wants to operate a food truck selling beef brisket and pulled pork. The truck would cost him about $75,000, and the truck would need to be fully equipped and functional before any product could be sold. He figures he'll make a lot of money doing it, because he's got some great BBQ sauce recipes, but if it doesn't work out, he can just sell the truck, eat the inventory, and get out of the business. The food truck would be best thought of as a... A. variable cost. B. fixed cost. C. sunk cost.

B. fixed cost.

Graphs like Figure 5-11 in the text sometimes show a Total Cost function and (Total) Variable Cost function. As Q increases, the vertical distance between the TC and VC curves... A. decreases. B. remains constant. C. increases.

B. remains constant.

When economists talk about "capital" in the production function, they are most likely thinking about... A. the financial resources needed to fund initial investment and operation of the enterprise. B. the equipment, machines, and tools needed to produce the firm's output.

B. the equipment, machines, and tools needed to produce the firm's output.

To maximize net benefits, a manager must ensure that for the last unit of any activity or production... A. the marginal benefit is greater than the marginal cost, MB > MC. B. the marginal benefit is equal to the marginal cost, MB = MC. C. the marginal benefit is less than the marginal cost, MB < MC. D. there is no marginal benefit left to obtain, MB = 0.

B. the marginal benefit is equal to the marginal cost, MB = MC.

At the point that maximizes total revenue along a demand function, the absolute value of own price elasticity is... A. 0. B. between 0 and 1. C. 1. D. greater than 1.

C. 1.

Your firm produces two similar products. Which of the following conditions implies that you should be concerned about lowering the price of one product because it might "cannibalize" sales of the other? A. Both products are very elastically demanded with respect to their own prices. B. Both products are very inelastically demanded with respect to their own prices. C. Cross price elasticity between the two is large and positive. D. Cross price elasticity between the two is large and negative.

C. Cross price elasticity between the two is large and positive.

Olivia leaves her old job, and starts up a new business as proprietor of a small bakery. She works long hours alongside her hired help, whom she pays a good hourly wage. She also incurs the costs of a renting her facilities and equipment, and of course the goods used in baking; after her expenses are paid, she takes home whatever is left, with some months being better than others - and it works out to about $4000 on average. What can we say about her economic profit versus her accounting profit. A. Her economic profit and her accounting profit are the same. B. Her economic profit is greater than her accounting profit. C. Her economic profit is less than her accounting profit.

C. Her economic profit is less than her accounting profit.

The text writes, Q = f(L) = F(K*,L) to describe a short-run production function. What does the * indicate? A. The * means that K has to be multiplied by L to be productive. B. The * means that K is the more important input in the production process. C. The * indicates that capital is fixed in the short run.

C. The * indicates that capital is fixed in the short run.

The text notes that sometimes the government will buy up (and discard) any surplus in the market resulting from a price floor - that is, the government provides a "price support." Does this action by the government increase or decrease the deadweight loss of an ordinary price floor? A. The government's price support action does not change the deadweight loss of the price floor. B. The government's price support action decreases the deadweight loss of the price floor. C. The government's price support action increases the deadweight loss of the price floor.

C. The government's price support action increases the deadweight loss of the price floor.

Suppose sport utility vehicles and small pick-up trucks are substitutes in production. If the price of sport utility vehicles increases, other things being the same, what should we expect in the market for small pick-up trucks? A. The demand curve for small pick-up trucks will shift to the left. B. The demand curve for small pick-up trucks will shift to the right. C. The supply curve for small pick-up trucks will shift to the left. D. The supply curve for small pick-up trucks will shift to the right.

C. The supply curve for small pick-up trucks will shift to the left.

Again, you've developed a linear demand model showing that QD = 50 - 4P + 2M, and P =15 and M = 25. Goods with a negative income elasticity of demand are called inferior goods. Goods with a positive income elasticity of demand are normal goods, which are sometimes divided into necessities (if income elasticity is less than 1) and luxuries (if income elasticity is greater than 1). This good, at this price and income level, is... A. an inferior good. B. a necessity. C. a luxury. D. We don't have enough information to determine whether this good is inferior, a necessity, or a luxury.

C. a luxury.

The text calls "the statistical analysis of economic data"... A. accounting. B. macroeconomics. C. econometrics. D. elasticity.

C. econometrics.

In general, the quantity of an activity or production level that maximizes total benefits will be... A. less than the level that maximizes net benefits. B. the same as the level that maximizes net benefits. C. greater than the level that maximizes net benefits. D. unrelated to the level that maximizes net benefits.

C. greater than the level that maximizes net benefits.

Consider a market in equilibrium. If the labor costs in the production process rise, we should expect... A. the demand curve to shift to the left, resulting in lower quantities sold and ultimately lower prices for the good. B. the demand curve to shift to the right, resulting in higher quantities sold and ultimately higher prices for the good. C. the supply curve to shift to the left, resulting in lower quantities sold, but ultimately high prices for the good. D. the supply curve to shift to the right, resulting in higher quantities sold, but ultimately lower prices for the good.

C. the supply curve to shift to the left, resulting in lower quantities sold, but ultimately high prices for the good.

From the text: what is the maximum amount you would pay for an asset that generates an income of $250,000 at the end of each of five years if the opportunity cost of using funds is 8 percent? (Your answer may differ slightly due to rounding.) A. $1,249,999 B. $1,157,407 C. $998,177 D. $850,728

C.$998,177

Suppose a competitive market's equilibrium price is $10, but a price ceiling is imposed and enforced at $7. Which of the following is a reasonable expectation of what the resulting "non-pecuniary price" would be? A. $0 B. $2 C. $3 D. $5

D. $5

Lisa has a "happiness meter" that she applies to time spent cleaning her garage. She is a net happiness maximizer, with one peculiar trait - she always works in one hour increments (never in half hours or 15 minute blocks of time, for example). The first hour spent cleaning the garage provides 80 units of happiness, and every hour after that provides additional happiness - but only at half the amount of the previous hour, so the second hour provides 40 units of happiness, the 3rd hour 20, and so forth. She has other things she could be doing and is acutely aware of the opportunity cost of her time, which doubles with each passing hour, starting with a cost of 1 unit of happiness for the first hour, 2 units for the second hour, 4 for the third, and so on. How many hours should she work on cleaning her garage? A. 1 B. 2 C. 3 D. 4 E. 5

D. 4

On a standard diagram (like those in the text) showing a demand curve, which of the following would shift demand to the right? A. An increase in the good's price. B. A decrease in the good's price. C. An increase in the price of a complement. D. A decrease in the price of a complement.

D. A decrease in the price of a complement.

Suppose you want to maximize your firm's profits and you're looking over a recent study reporting own price elasticity of demand for your product. Which of the following is an accurate way of interpreting the report? A. If the report finds that demand is elastic, you should plan to reduce the price of your product - the decrease in price will increase total revenue and is certain to increase profit. B. If the report finds that demand is elastic, you should plan to increase the price of your product - the increase in price will increase total revenue and is certain to increase profit. C. If the report finds that demand is inelastic, you should plan to decrease the price of your product - the decrease in price will increase total revenue and is certain to increase profit. D. If the report finds that demand is inelastic, you should plan to increase the price of your product - the increase in price will increase total revenue and is certain to increase profit.

D. If the report finds that demand is inelastic, you should plan to increase the price of your product - the increase in price will increase total revenue and is certain to increase profit.

If demand is given by P = 5 - Q and the price of the good rises from $2 to $3, what happens to consumer surplus? A. It decreases by $1. B. It decreases by more than $1 but less than $2. C. It decreases by $2. D. It decreases by more than $2.

D. It decreases by more than $2.

A prospective MBA student estimates that her program of study will cost $40,000, but that she will increase her earnings by $54,000 over the course of her expected 25 year career. She then uses a present value calculator she found on the Internet, which - based on a 10% interest rate - finds that the present value of the program's cost is $38,200, but that the present value of her increase earnings is only $20,300, and it recommends that she not enroll in this program. She concludes that the calculator must be wrong, because even though present value calculations deflate the future costs and the future incomes, they should decrease by the same percent - which they clearly didn't do. She decides to go ahead and enroll in the program, because the $54,000 benefit is greater than the $40,000 cost. Did she make the right choice for the right reason? A. Yes, she should enroll, both because the benefit exceeds the cost, and because the Internet present value calculator cannot be correct. B. Yes, she should enroll because the benefit exceeds the cost - but she's wrong about the present value calculator, which may in fact be correct. C. Yes, she should enroll, but not for any of the reasons given

D. No, she should not enroll; it's entirely possible for the present value calculator to be correct, and it does not matter that the $54,000 benefit over her career is greater than the cost of the program.

You want to produce 30,000,000 units of output. Your production function is Q = aK + bL. L costs twice as much as K per unit. If a = 2 and b = 1, how much L should you hire? A. 30,000,000 B. 15,000,000 C. 10,000,000 D. None.

D. None.

In Siloam Springs, Arkansas, you pay 9.5% sales tax on the purchases of ordinary goods from a store. The text says... A. This is an excise tax, and it shifts the supply curve to the right. B. This is an excise tax, and it shifts the supply curve to the left. C. This is an ad valorem tax, and it shifts the supply curve to the right. D. This is an ad valorem tax, and it shifts the supply curve to the left.

D. This is an ad valorem tax, and it shifts the supply curve to the left.

An "inferior good" has which of the following characteristics? A. When its price increases, more is demanded. B. When its price increases, less is demanded. C. When income increases, more is demanded. D. When income increases, less is demanded.

D. When income increases, less is demanded.

Suppose that demand for good X is given by an equation: QDX = 12,000 - 3PX + 4PY - 1M + 2A, where PY is the price of Y, M is income, and A is advertising. What can we tell from this equation? A. X and Y are complements and X is a normal good. B. X and Y are substitutes and X is a normal good. C. X and Y are complements and X is an inferior good. D. X and Y are substitutes and X is an inferior good.

D. X and Y are substitutes and X is an inferior good.

You produce good X and another firm produces good Y. Last year you raised your price 10% and your quantity sold dropped 6%, and Y's price didn't change. This year, you kept your price the same but Y raised their price 10% - and your sales dropped 4%. Assuming other conditions remained the same, what can we conclude? A. X is elastically demanded, and X and Y are substitutes. B. X is inelastically demanded, and X and Y are substitutes. C. X is elastically demanded, and X and Y are complements. D. X is inelastically demanded, and X and Y are complements.

D. X is inelastically demanded, and X and Y are complements.

A university's football stadium currently seats 26,000, but end-zone seating could be added to expand seating capacity to 34,000. In the language of the textbook, the decision to add (or not add) end-zone seating is best viewed as... A. a discrete decision. B. a marginal decision. C. a continuous decision. D. an incremental decision.

D. an incremental decision.

On the inverse demand diagram described in the last question, suppose the inverse demand function is a vertical line. In this case, demand would be... A. perfectly elastic, and the good is most likely an extreme luxury. B. perfectly elastic, and the good is most likely an extreme necessity. C. perfectly inelastic, and the good is most likely an extreme luxury. D. perfectly inelastic, and the good is most likely an extreme necessity.

D. perfectly inelastic, and the good is most likely an extreme necessity.

Complete this famous quotation from Adam Smith's The Wealth of Nations. "It is not out of the benevolence of the butcher, the brewer, or the baker, that we expect our dinner, but from... A. the collective obligation of civilized society to provide for each and every citizen." B. the coercive pressure of society's institutions, not the least of which is the government itself." C. the good will of those who actually care for us and love us, those who indeed act on our behalf and for our welfare." D. their regard to their own interest."

D. their regard to their own interest."

Let's make sure we remember the big "Six Principles" that are the focus of Chapter 1. These principles say that effective managers... understand markets; can identify goals and constraints; can use marginal analysis; recognize the nature and importance of profits; recognize the time value of money, and.... Which one is missing? A. understand the appropriate role of government in the marketplace. B. grasp the "big picture" difference between socialism and capitalism. C. vote Republican. D. understand incentives.

D. understand incentives.

Implicit costs are: a) "payments" for self-employed resources. b) always greater in the short run than in the long run. c) equal to total fixed costs. d) comprised entirely of variable costs.

a) "payments" for self-employed resources.

Moped, Inc. purchased machinery at a cost of $44,000 on January 1, 2017. The expected useful life is 5 years and the asset is expected to have salvage value of $4,000. Moped depreciates its assets using the double-declining balance method.What is the firm's depreciation expense for the year ended December 31, 2017? a) $17,600 b) $4,000 c) $12,000 d) $8,800

a) $17,600

Val's travel budget for October was $720, based on her plan to drive 3,000 miles at a cost of $0.24 per mile. During October, she actually drove 2,800 miles at a total cost of $700. A flexed budget performance report would show a variance of: a) $28 U. b) $30 U. c) $20 F. d) $50 F.

a) $28 U.

On January 31, an entity's balance sheet showed net assets of $3,075 and liabilities of $675. Stockholders' equity on January 31 was: a) $3,075 b) $2,400 c) $3,750 d) $675

a) $3,075

If a firm's debt ratio was 25%, its debt/equity ratio would be: a) 33.33%. b) 25%. c) 75%. d) 50%.

a) 33.33%.

ABC Company's sales are $100,000, fixed costs are $50,000, and variable costs are $30,000. ABC Company's contribution margin ratio is: a) 70%. b) 20%. c) 50%. d) 30%.

a) 70%.

Which of the following is true regarding notes receivables? a) A note is a more formal document than an account receivable. b) A notes receivable is always a current asset. c) A notes receivable is always a long-term asset. d) A note is a less formal document than an account receivable.

a) A note is a more formal document than an account receivable.

Which of the following is the correct balance sheet presentation for current assets? a) Accounts receivable, inventories, prepaid expenses, other current assets. b) Cash equivalents, cash, other current assets, accounts receivable. c) Cash, inventories, account receivables, prepaid expenses. d) Marketable securities, cash, notes receivable, prepaid expenses.

a) Accounts receivable, inventories, prepaid expenses, other current assets.

Which of the following is not a stockholders' equity account? a) Accumulated depreciation. b) Capital stock. c) Retained earnings. d) Common stock. e) Paid-in-capital in excess of par.

a) Accumulated depreciation.

Which of the terms is not used to identify owners' equity or stockholders' equity? a) Additional-paid-in-retained earnings. b) Partner's capital. c) Proprietor's capital. d) Paid-in-capital and retained earnings.

a) Additional-paid-in-retained earnings.

Why did Sowell stress that market systems are both profit and loss systems? a) Losses are an important signal to tell businesses which products/services not to produce. b) To appease politicians who don't like businesses. c) Businesses need losses to offset taxes. d) All businesses generally make profits and charge too much for their products/services.

a) Losses are an important signal to tell businesses which products/services not to produce.

Which of the following is not a topic that is likely to be discussed as a significant accounting policy? a) Method of estimating uncollectible accounts receivable. b) Earnings per share of common stock calculation details. c) Depreciation method. d) Inventory valuation method.

a) Method of estimating uncollectible accounts receivable.

An Accounts Payable normally results from which of the following transactions? a) Purchasing goods and services from suppliers on credit. b) Purchasing property, plant and equipment on credit. c) Purchasing accounts for cash. d) All of the above.

a) Purchasing goods and services from suppliers on credit.

Critics of the minimum wage argue that an increase in the minimum wage rate above the equilibrium rate of a purely competitive labor market would: a) Reduce employment (increase unemployment) in the low-skilled labor market. b) increase firms' demand for labor. c) decrease the supply of labor. d) cause firms to substitute labor for capital.

a) Reduce employment (increase unemployment) in the low-skilled labor market.

Which of the following is a universally accepted measure of profitability? a) Return on investment. b) Return on liabilities. c) Return on retained earnings. d) All of these.

a) Return on investment.

According to Sowell, how did Margaret Thatcher respond to Mikhail Gorbachev's questions: "how do you see to it that people get food?" a) She indicated, "She {the government} didn't, prices did that." b) She indicated they use a system based on greatest need as determined by parliament. c) She indicated a surplus had been acquired from countries with abundance. d) She said, "A leader's greatest responsibility is to feed their people."

a) She indicated, "She {the government} didn't, prices did that."

The price consumers pay for waiting in line when price ceilings are imposed is called: a) The nonpecuniary price. b) Pecuniary price c) Market disruption price. d) Inefficient price.

a) The nonpecuniary price.

Which of the following was not one of the reasons for the FASB's decision to undertake the Conceptual Framework project? a) To provide detailed, industry-specific authoritative guidance for the accounting and financial reporting of complex business transactions. b) To provide a structure or framework of financial accounting concepts. c) To facilitate the FASB's efforts in developing accounting and reporting guidance by providing a common foundation and basic reasoning on which to consider merits of alternatives. d) To describe concepts that will underlie guidance on future accounting practices and in due course serve as a basis for evaluating existing guidance and practices.

a) To provide detailed, industry-specific authoritative guidance for the accounting and financial reporting of complex business transactions.

What is the difference between a direct and indirect distribution channel? a) a direct distribution channel involves selling products directly to customers, while and indirect involves selling products through intermediaries b) indirect involves selling products directly to customers, while direct involves selling products through intermediaries c) direct involves smaller amounts of money than indirect d) indirect involves more stakeholders than direct

a) a direct distribution channel involves selling products directly to customers, while and indirect involves selling products through intermediaries

A leveraged buyout refers to: a) a firm goes heavily into debt in order to obtain the funds to purchase the shares of the public stockholders and thus take the firm private. b) one firm trades its stock for the stock of another firm. c) one firm issues stock to take over another firm. d) one firm pays cash for the shares of a takeover firm's shares.

a) a firm goes heavily into debt in order to obtain the funds to purchase the shares of the public stockholders and thus take the firm private.

What is the difference between a marketing plan and a marketing strategy? a) a marketing plan is a detailed document outline specific tactics to achieve a marketing goal, while marketing strategy is a broad plan outline the overall approach to marketing b) a marketing plan is a broad plan outline the overall approach to marketing, while a marketing strategy is a detailed document outline specific tactics to achieve a marketing goal c) a marketing plan and a marketing strategy have the same meaning and can be used interchangeably d) a marketing plan and a marketing strategy are not important for a company to succeed

a) a marketing plan is a detailed document outline specific tactics to achieve a marketing goal, while marketing strategy is a broad plan outline the overall approach to marketing

What is the difference between a product line a product mix? a) a product line is a group of related products that are sold by a company, while a product mix is the total number of products sold by a company b) a product mix is a group of related products that are sold by a company, while a product line is the total number of products sold by a company c) a product line is the total number of products sold by a company while a product mix is a group of unrelated products sold by a comapny d) a product mix and a product line have the same meaning and can be used interchangeably

a) a product line is a group of related products that are sold by a company, while a product mix is the total number of products sold by a company

A transaction that is likely to cause an increase in a current liability is: a) accrual of interest expense. b) payment of accrued wages. c) accrual of bad debts expense. d) depreciation of equipment.

a) accrual of interest expense.

The ethical concept of integrity means that an individual must: a) attempt to be honest and forthright in dealings and communications with others. b) report to a supervisor any violation of the code of conduct of her company that is observed. c) sign a pledge to abide by all laws and regulations. d) read, understand, and agree to follow all provisions of her employer's code of conduct.

a) attempt to be honest and forthright in dealings and communications with others.

Cost of Goods Manufactured can be computed as: a) beginning balance of work in process + raw materials used + direct labor costs incurred + manufacturing overhead costs applied - ending balance of work in process. b) beginning balance of work in process + raw materials purchased + direct labor costs incurred + manufacturing overhead costs applied - ending balance of work in process. c) ending balance of work in process + raw materials used + direct labor costs incurred + manufacturing overhead costs applied - beginning balance of work in process. d) ending balance of work in process + raw materials purchased + direct labor costs incurred + manufacturing overhead costs applied - beginning balance of work in process.

a) beginning balance of work in process + raw materials used + direct labor costs incurred + manufacturing overhead costs applied - ending balance of work in process.

How can a company use market segmentation to better target its customers? a) by identifying the unique needs and preferences of different customer groups b) by creating a single product or service that meets the needs of all custoemrs c) by setting prices based on the income levels of different customer groups d) by using social media to reach a wider audience of potential customers

a) by identifying the unique needs and preferences of different customer groups

How can a company use customer relationship management to improve its marketing effectiveness? a) by using data to segment customers and tailor marketing messages to their needs and preferences b) by offering discounts and promotions to customers who make repeat purchases c) by using social media to engaged with customers d) all of the above

a) by using data to segment customers and tailor marketing messages to their needs and preferences

The kinked-demand curve of an oligopolist is based on the assumption that: a) competitors will follow a price cut but ignore a price increase. b) competitors will match both price cuts and price increases. c) competitors will ignore a price cut but follow a price increase. d) there is no product differentiation.

a) competitors will follow a price cut but ignore a price increase.

Which activity of the management planning and control process occurs in each phase of the cycle? a) decision making. b) managing. c) controlling. d) planning.

a) decision making.

The three components of product costs are: a) direct material, direct labor, manufacturing overhead. b) direct labor, manufacturing overhead, indirect material. c) manufacturing overhead, indirect material, indirect labor. d) direct material, supervisor salaries, selling expenses.

a) direct material, direct labor, manufacturing overhead.

Which of the following items would be reported on the Statement of Cost of Goods Manufactured? a) ending work in process inventory. b) cost of goods sold. c) beginning finished goods inventory. d) contribution margin.

a) ending work in process inventory.

An audit conducted in accordance with generally accepted auditing standards includes each of the following except: a) evaluation of the efficiency and effectiveness of management. b) assessment of the accounting principles used and significant estimates made by management. c) examination, on a test basis, of evidence supporting the amounts and disclosures in the financial statements. d) planning and performance of the audit to obtain reasonable assurance that the financial statements are free of material misstatements.

a) evaluation of the efficiency and effectiveness of management.

The primary difference between absorption costing and direct costing is the treatment of: a) fixed manufacturing overhead. b) direct labor costs. c) variable manufacturing overhead. d) direct material costs.

a) fixed manufacturing overhead.

Opportunity costs are: a) foregone benefits. b) included in cost of goods sold. c) included in inventory. d) sunk costs.

a) foregone benefits.

A credit entry will: a) increase the balance of a revenue account. b) increase the balance of an expense account. c) always increase the account balance. d) always decrease the account balance.

a) increase the balance of a revenue account.

The price/earnings ratio: a) is a measure of the relative expensiveness of a firm's common stock. b) does not usually change by more than 1.0 (e.g. 8.2 to 9.2) during the year. c) is calculated by dividing the earnings multiple by net income. d) can be used to determine the cash dividend to be received during the year.

a) is a measure of the relative expensiveness of a firm's common stock.

The distinction between a current asset and other assets: a) is based on when the asset is expected to be converted to cash, or used to benefit the entity. b) is based on the ability to determine the current fair value of the asset. c) is based on amounts that will be paid to other entities within a year. d) is based on how long the asset has been owned.

a) is based on when the asset is expected to be converted to cash, or used to benefit the entity.

A balanced scorecard framework is integrated through four key perspectives. The presentation of these key perspectives on a balanced scorecard from the lowest level perspective to the highest level perspectives is: a) learning and growth, internal business process, customer, financial. b) customer, financial, internal business process, learning and growth. c) financial, customer, learning and growth, internal business process. d) internal business process, learning and growth, customer financial.

a) learning and growth, internal business process, customer, financial.

Which of the following is a key metric used in strategic marketing planning? a) market share b) revenue growth c) brand awareness d) customer satisfaction

a) market share

What is the difference between consumer needs and wants? a) needs are more basic than wants and are essential for survival, while wants are not b) wants are more basic than needs and are essential for survival, while needs are not c) needs and wants are interchangeable terms and have the same meaning d) needs and wants have no impact on consumer behavior

a) needs are more basic than wants and are essential for survival, while wants are not

An economy is producing at the least-cost rate of production when: a) price and the minimum average total cost are equal. b) marginal cost is greater than average total cost. c) marginal revenue is greater than price. d) price and marginal revenue are equal.

a) price and the minimum average total cost are equal.

What is the difference between primary and secondary data? a) primary is collected by the company for a specific purpose, while secondary data is data that has been collected by others for a different purpose b) primary is data that is collected by competitors, while secondary data is collected by the company. c) primary data is collected through surveys, while secondary data is collected through observation d) primary data is collected through focus groups, while secondary data is collected through interview

a) primary is collected by the company for a specific purpose, while secondary data is data that has been collected by others for a different purpose

what is the difference between a push and a pull promotional strategy? a) push involves using advertising and promotions to create demand for a product or service, while pull involves using personal selling and word-of-mouth to create demand b) pull involves using advertising and promotions to create demand for a product or service, while pull involves using personal selling and word-of mouth to create demand c) they have the same meaning and can be used interchangeably d) push involves offering discounts and promotions to customers, while pull involves using social media to engage with customers

a) push involves using advertising and promotions to create demand for a product or service, while pull involves using personal selling and word-of-mouth to create demand

what is the difference between quantitative and qualitative research methods? a) quantitative research is based on numbers and statistics, while qualitative research is baed on non-numerical data such as words and images b) quantitative research is more reliable than qualitative research c) qualitative research is more valid than qualitative research d) both quantitative and qualitative research methods have the same level of accuracy

a) quantitative research is based on numbers and statistics, while qualitative research is baed on non-numerical data such as words and images

A journal entry recording an accrual: a) results in a better matching of revenues and expenses. b) will affect balance sheet accounts only. c) will involve a debit or credit to cash. d) will most likely include a debit to a liability account.

a) results in a better matching of revenues and expenses.

Which of the following budgets must be completed before a budgeted income statement can be prepared? a) sales, cost of goods sold, and operating expense. b) sales, cash, and balance sheet. c) sales, cash, and cost of goods sold. d) sales, operating expense, and cash.

a) sales, cost of goods sold, and operating expense.

Income from operations is: a) sometimes used in the ROI calculation. b) usually calculated after income tax expense. c) sometimes called the "bottom line." d) usually used in the ROE calculation.

a) sometimes used in the ROI calculation.

An unqualified auditors' opinion about an entity's financial statements: a) states that they are presented in conformance with U.S. generally accepted accounting principles. b) is a clean bill of health. c) means that all of the entity's transactions during the audited period were checked out. d) guarantees that the entity was not involved in or the victim of any fraudulent activities during the audited period.

a) states that they are presented in conformance with U.S. generally accepted accounting principles.

Standards are most appropriately used to: a) support the planning and control processes of the firm. b) calculate the unit cost of a product or service. c) penalize workers and managers who do not meet them. d) reward workers and managers who meet them.

a) support the planning and control processes of the firm.

The most powerful corporate governance legislation to date has been: a) the Sarbanes-Oxley Act (SOX) of 2002. b) the regulation of inventory management practices by the SEC. c) the creation of the American Institute of Certified Public Accountants. d) Corporate Ethics Code of 2007.

a) the Sarbanes-Oxley Act (SOX) of 2002.

Unions benefit ____________ but hurt ______________. a) the union members who are employed, other union and non-union workers who are not employed b) union employers, non-union employers c) professionals (white collar workers), non-professional (blue collar workers) d) minorities, women

a) the union members who are employed, other union and non-union workers who are not employed

what is the purpose of a marketing budget? a) to allocate resources to marketing activities in a way that maximized the company's return on investment b) to set financial targets for the company c) to identify potential customers for the company's products or services d) to provide a detailed analysis of the company's competition

a) to allocate resources to marketing activities in a way that maximized the company's return on investment

What is the purpose of a marketing dashboard? a) to track and analyze marketing performance metrics b) to communicate marketing goals and strategies to stakeholders c) to provide a detailed analysis of the company's competition d) to identify potential customers for the company's products or services

a) to track and analyze marketing performance metrics

Management's statement of responsibility: a) usually refers to the company's system of internal controls. b) includes a disclaimer of responsibility for the level of the P/E ratio of the company's common stock. c) gives the president of the company an opportunity to explain why profits changed. d) emphasizes that the auditors are responsible for the financial statements.

a) usually refers to the company's system of internal controls.

The contribution margin ratio always decreases when the: a) variable cost increase and the selling price remains constant. b) break-even point decreases. c) selling price increases and the variable costs remain constant. d) fixed expenses increase.

a) variable cost increase and the selling price remains constant.

Financial statement ratios support informed judgments and decision making most effectively: a) when the trend of entity data is compared to the trend of industry data. b) when viewed for a single year. c) when compared to an industry average for the most recent year. d) when viewed as a trend of entity data.

a) when the trend of entity data is compared to the trend of industry data.

Economies and diseconomies of scale explain: a) why the firm's long-run average total cost curve is U-shaped. b) the distinction between fixed and variable costs. c) why the firm's short-run marginal cost curve cuts the short-run average variable cost curve at its minimum point. d) the profit-maximizing level of production.

a) why the firm's long-run average total cost curve is U-shaped.

A shipping firm spends $10,000 in attorney and consultant fees negotiating a $30,000 fuel contract with a supplier. The transactions costs of procuring the fuel is best thought of as... a. $10,000 b. $20,000 c. $30,000 d. $40,000

a. $10,000

a company manufacturers and sells a product with a selling price of $100 per unit the variable cost per unit is $60 and the fixed costs are 200 and $1000 the company has a tax rate of 30% what is the company's contribution margin per unit a. $40 b. $60 c. $100 d. $160

a. $40

you are a financial analyst for a large multinational corporation that is considering a new project in a foreign country the project requires an investment of $10 million and is expected to generate annual cash flow of $3 million for the next 10 years the required rate of return for the project is 15% and the exchange rate is currently one USD = 100 foreign currency units what is the nearest net present value of this project in USD should the company invest in this project a. $5,100,000 yes b. $3,300,000 yes c. $4,700,000 yes negative $3,300,000 no

a. $5,100,000 yes

What is the difference between a high involvement and a low involvement purchase? a. A high involvement purchase is a purchase that is important to the consumer and involves the significant investment of time and effort a low improvement purchase is a purchase that is not important to the consumer and does not involve a significant investment of time and effort b. a high involvement purchase is a purchase that is not important to the consumer and does not involve a significant investment of time and effort a low involvement purchase is a purchase that is important to the consumer and involves a significant investment of time and effort c. a high involvement purchase is a purchase that is important to the consumer and involves a significant investment of money a low involvement purchase is a purchase that is not important to the consumer and does not involve a significant investment of money d. a high involvement purchase is a purchase that is not important to the consumer and does not involve a significant investment of money a low involvement purchase is a purchase that is important to the consumer and involves a significant investment of money

a. A high involvement purchase is a purchase that is important to the consumer and involves the significant investment of time and effort a low improvement purchase is a purchase that is not important to the consumer and does not involve a significant investment of time and effort

Two industries each have four firms. Industry A has 4 firms, each with a market share of 25%. Industry B has 4 firms, but one firm has 70% market share and the other three each have 10%. Which of the following statements is correct regarding concentration measurement? a. Both industries have the same four firm concentration ratio. b. Industry A has the higher four firm concentration ratio. c. Industry B has the higher four firm concentration ratio.

a. Both industries have the same four firm concentration ratio.

which of the following methods is used to determine the target profit for a given period a. Contribution margin ratio method b. unit sales method c. absorption costing method d. none of the above

a. Contribution margin ratio method

in a special order decision which of the following costs are relevant a. Only variable costs b. only fixed costs c. both variable and fixed costs d. none of the above

a. Only variable costs

what is the purpose of the swot analysis and how can it be used in marketing planning a. a SWOT analysis is a tool for identifying a company's strengths weaknesses opportunities and threats it can be used in marketing planning to help a company develop a marketing strategy that takes advantage of its strengths addresses its weaknesses and responds to its opportunities and threats b. a SWOT analysis is a tool for identifying a company's financial performance it can be used in marketing planning to set financial targets for the marketing strategy c. a SWOT analysis is a tool for identifying a company's customer base it can be used in marketing planning to segment the market and target specific consumer groups d. a SWOT analysis is a tool for identifying a company's competitors it can be used in marketing planning to develop a strategy to copy it's competitors products or services

a. a SWOT analysis is a tool for identifying a company's strengths weaknesses opportunities and threats it can be used in marketing planning to help a company develop a marketing strategy that takes advantage of its strengths addresses its weaknesses and responds to its opportunities and threats

what is the difference between a push and pull supply chain strategy a. a push supply chain strategy involves producing goods based on anticipated demand and pushing them through the supply chain to the consumer a pool supply chain strategy involves producing goods in response to customers demands and pulling them through the supply chain to the consumer b. a pole supply chain strategy involves producing goods based on anticipated demand and pushing them through the supply chain to the customer a push supply chain strategy involves producing goods in response to customer demand and pulling them through the supply chain to the customer c. a push supply chain strategy and a poor supply chain strategy have the same meaning and can be used interchangeably d. a push supply chain strategy involves producing goods and storing them in warehouses while a pool supply chain strategy involves producing goods and delivering them directly to the customer

a. a push supply chain strategy involves producing goods based on anticipated demand and pushing them through the supply chain to the consumer a pool supply chain strategy involves producing goods in response to customers demands and pulling them through the supply chain to the consumer

which of the following financial ratio measures a company's ability to pay its short term obligations with its current assets a. acid test ratio b. debt to equity ratio c. return on equity d. price earning ratio

a. acid test ratio

which of the following is a key element of effective conflict resolution a. active listening and collaboration b. avoidance and denial c. assertiveness and aggression d. delegating their resolution to a third party

a. active listening and collaboration

which of the following methods is used to analyze the profitability of a company's product lines or customer segments a. activity based costing b. traditional casting c. target costing d. life cycle costing

a. activity based costing

which of the following is a key consideration in designing an effective organizational structure a. aligning the structure with the organization's strategy and goals b. reducing the cost of Labor c. increasing the number of hierarchal levels d. the focusing on individual job responsibilities rather than collaboration

a. aligning the structure with the organization's strategy and goals

A high Rothschild Index could be interpreted as meaning... a. an industry has few competitors, or there is significant brand loyalty if there are many competitors. b. an industry has many competitors, but there is little brand loyalty. c. the industry is important to consumers, but firms in the industry have little market power to control price. d. demand for industry's product is low relative to the number of firms in the market.

a. an industry has few competitors, or there is significant brand loyalty if there are many competitors.

which of the following financial statements reports a company's assets liabilities and equity at a specific point in time a. balance sheet b. income statement c. statement of cash flows d. statement of changes in equity

a. balance sheet

a manufacturing company is considering the purchase of new machinery to increase its production capacity the company has limited funds and is trying to decide whether to purchase the machinery outright or lease it you've been hired as a consultant to help the company make an informed decision which of the following financing options would be most appropriate for the manufacturing company to purchase the machinery a. bank loan b. trade credit c. factoring d. Angel investment

a. bank loan

which of the following conflict resolution strategies is most appropriate for situations where both parties need to achieve their objective a. collaboration b. compromise c. avoidance d. competition

a. collaboration

you're the HR manager of a large corporation that is struggling to retain top talent due to lack of career growth opportunities which of the following strategies would be most effective in developing a career growth plan a. collaboration with employees to identify career goals and development opportunities b. the use of standardized career paths and development programs for all employees c. the delegation of career development to individual employees d. the focus on external recruitment to bring in new talent

a. collaboration with employees to identify career goals and development opportunities

a retail chain is experiencing a decline in sales and profitability due to increased competition and changing customer preferences the company has decided to implement a cost cutting program to improve its financial performance you have been hired as a consultant to help the company identify areas where cost cutting can be achieved without compromising the quality of products and services what steps should the retail chain take to identify areas where cost cutting can be achieved a. conduct A detailed analysis of the company's cost structure and identify areas of inefficiency b. evaluate the quality of products and services and identify areas where improvements can be made c. review the supply chain and logistics process to identify areas for improvement d. all of the above

a. conduct A detailed analysis of the company's cost structure and identify areas of inefficiency

which of the following methods is used to determine the break even point in sales dollars a. contribution margin ratio method b. unit sales method c. Absorption costing method d. none of the above

a. contribution margin ratio method

which of the following methods is used to determine the sales price necessary to achieve a target profit a. contribution margin ratio method b. unit sales method c. absorption costing method d. none of the above

a. contribution margin ratio method

which of the following methods is used to determine the sales volume necessary to achieve a target profit a. contribution margin ratio method b. unit sales method c. absorption costing method d. none of the above

a. contribution margin ratio method

which of the following financial ratios measures the company's ability to meet its short term obligations a. current ratio b. debt to equity ratio c. return on equity d. price earning ratio

a. current ratio

A healthcare organization is planning to invest in new technology to improve patient care and outcomes the organization has limited funds and is trying to decide whether to finance it investment through debt or equity you have been hired as a consultant to help the organization make an informed decision which of the following financing options would be most appropriate for the healthcare organization to finance the investment a. debt financing b. equity financing c. public private partnership d. grants and subsidies

a. debt financing

which of the following costs are considered relevant in a make or buy decision a. direct materials and direct labor only b. direct materials direct labor and variable overhead c. direct materials direct labor variable overhead and fixed overhead d. all manufacturing costs

a. direct materials and direct labor only

you are a portfolio manager for a large investment firm that is considering an investment in a startup technology company the company has developed a new software application that is expected to revolutionize the industry however the company is not yet profitable and has a high degree of risk which of the following strategies would be most appropriate to manage the risk of this investment a. diversification b. hedging c. short selling d. margin trading

a. diversification

which of the following methods of calculating the cost of equity is most appropriate for a firm that pays dividends a. dividend discount model b. capital asset pricing model c. weighted average cost of capital d. net present value

a. dividend discount model

what is the difference between exploratory and conclusive research a. exploratory research is research that is conducted to gain a general understanding of a topic while conclusive research is research that is conducted to answer specific questions or test hypotheses b. exploratory research is research that is conducted to test hypotheses while conclusive research is research that is conducted to gain a general understanding of a topic c. exploratory research and conclusive research have the same meaning and can be used interchangeably d. exploratory research is research that is conducted by competitors while conclusive research is research that is conducted by a company for a specific purpose

a. exploratory research is research that is conducted to gain a general understanding of a topic while conclusive research is research that is conducted to answer specific questions or test hypotheses

a startup company is planning to enter a new market with a disruptive technology that has the potential to revolution and eyes the industry you've been hired as a consultant to help the company develop a strategy that will enable it to achieve a dominant position in the market which of the following strategies would be most effective for the startup company to establish itself as a dominant player in the market a. focus on product differentiation and innovation b. pursue a low cost strategy to capture market share c. form strategic partnerships with established players in the industry d. all of the above

a. focus on product differentiation and innovation

which of the following statements is true about the margin of safety a. it is the amount by which sales exceeds fixed costs b. it is the amount by which sales exceeds variable costs c. is the amount by which sales exceed total costs d. it is the amount by which total costs exceed sales

a. it is the amount by which sales exceeds fixed costs

which of the following statements is true about the contribution margin a. it is the amount by which sales exceeds variable costs b. it is the amount by which sales exceeds total costs c. it is the amount by which total cost exceeds sales d. it is the amount by which fixed costs exceed variable costs

a. it is the amount by which sales exceeds variable costs

which of the following statements is true about the break even point a. it is the point at which total revenue equals total cost b. it is the point at which variable cost equals fixed cost c. it is the point at which total contribution margin equals total fixed cost d. it is the point at which variable costs per unit equal fixed cost per unit

a. it is the point at which total revenue equals total cost

which of the following statements is true about process costing a. it is used for goods that are produced in a continuous flow b. it is used for goods that are produced in small batches c. it is used for goods that are customized and made to order d. it is used for goods that are mass produced and identical or very similar

a. it is used for goods that are produced in a continuous flow

which of the following factors are considered when setting transfer prices a. market price cost and negotiation b. variable cost and fixed cost c. target profit and break even analysis d. marginal cost and average cost

a. market price cost and negotiation

Contracts for input procurement are more attractive when... a. market transactions costs are high and the contract environment involves little uncertainty. b. market transactions costs are high and the contract environment involves much uncertainty. c. market transactions costs are low and the contract environment involves little uncertainty. d. market transactions costs are low and the contract environment involves much uncertainty.

a. market transactions costs are high and the contract environment involves little uncertainty.

which of the following is a key element of effective service management a. meeting or exceeding customer expectations b. maximizing internal efficiency at all costs c. focusing solely on the bottom line d. minimizing the amount of time spent interacting with customers

a. meeting or exceeding customer expectations

what is the role of motivation and consumer behavior a. motivation is a driving force behind all consumer behavior it is the reason why consumers make certain choices and take certain actions b. motivation has a minor impact on consumer behavior consumers make choices and take actions based on rational analysis of their needs and preferences c. motivation influences consumer behavior but is not the only factor consumers also make choices based on factors such as their social environment and previous experience d. motivation has no impact on consumer behavior

a. motivation is a driving force behind all consumer behavior it is the reason why consumers make certain choices and take certain actions

you are the CFO of a manufacturing company that is considering an investment in new production equipment the equipment has a cost of $5 million and is expected to generate annual cash flows of 1.5 million for the next five years the equipment has a salvage value of $500,000 at the end of the fifth year the company's cost of capital is 10% which of the following capital budgeting techniques would be most appropriate to evaluate this investment a. net present value b. internal rate of return c. payback period d. Profitability index

a. net present value

you are the COO of a large corporation that is struggling to keep up with changing industry trends and competition which of the following organizational structure designs would be most effective in adapting to changing industry trends a. of flexible and adaptable structure that can respond to changing demands b. a rigid and hierarchical structure that maintains the status quo c. the delegation of structure designed to lower level employees d. the use of standardized structure for all organizations

a. of flexible and adaptable structure that can respond to changing demands

In a sell or process further decision which of the following costs are relevant a. only additional costs beyond the split off point b. only joint costs up to the split off point c. both joint costs up to the split off point and additional costs beyond the split off point d. none of the above

a. only additional costs beyond the split off point

which of the following financial ratios measures the company's profitability relative to its assets a. return on assets b. return on equity c. price earning ratio d. debt to equity ratio

a. return on assets

The more complex the contracting environment, the more likely it is that the optimal length of the contract, time-wise, is... a. short. b. long.

a. short.

which of the following is a primary factor that influences team performance a. team composition b. team size c. team location d. team tenure

a. team composition

which of the following is a key element of effective control a. the ability to monitor and adjust performance in real time b. the focus on rigid adherence to established processes and procedures c. the use of standardized control measures for all situations d. the delegation of control to lower level employees

a. the ability to monitor and adjust performance in real time

which of the following is a key element of effective capacity planning a. the ability to respond quickly to changes in demand b. the maximization of production capacity at all times c. the delegation of capacity planning responsibilities to lower level employees d. the focus on minimizing capacity utilization costs

a. the ability to respond quickly to changes in demand

you're the HR manager of a large corporation that is struggling to retain top talent due to lack of career growth opportunities which of the following compensation strategies would be most effective in retaining top talent a. the alignment of compensation with employee performance and organizational goals b. the use of standardized compensation packages for all employees c. the focus on minimizing compensation costs d. the delegation of compensation management to lower level employees

a. the alignment of compensation with employee performance and organizational goals

which of the following is a key element of effective service management a. the alignment of service delivery with customer needs and expectations b. the minimization of customer contact with service providers c. the use of technology to automate service delivery processes d. the focus on maximizing internal efficiency

a. the alignment of service delivery with customer needs and expectations

a company that produces organic health foods is planning to expand its product line to include vegan protein bars how can the company use environment scanning to identify potential opportunities and threats in the market a. the company can conduct a PEST analysis to identify political economic social and technological factors that may impact the market for vegan protein bars b. the company can use SWOT analysis to identify its own strengths weaknesses opportunities and threats in the market for vegan protein bars c. the company can conduct a competitive analysis to identify the strengths and weaknesses of its competitors in the market for vegan protein bars

a. the company can conduct a PEST analysis to identify political economic social and technological factors that may impact the market for vegan protein bars

a home improvement retailer is considering expanding into a new geographic market how can the company use market research to inform its marketing strategy in the new market a. the company can conduct a consumer survey to gather information on the local consumer preferences and purchasing behaviors as well as what types of home improvement products are most in demanded in new market b. the company can conduct a competitive analysis to identify what home improvement retailers are already present in the new market and what their key strengths and weaknesses are c. The company can segment the new market based on factors such as age income and lifestyle to develop more targeted marketing campaigns for different consumer groups

a. the company can conduct a consumer survey to gather information on the local consumer preferences and purchasing behaviors as well as what types of home improvement products are most in demanded in new market

a luxury hotel chain is considering launching a new loyalty program to increase consumer retention and drive repeat business how can the company use market research to inform the development of the loyalty program a. the company can survey its existing customers to gather information on what features and rewards they value most and a loyalty program and what would incentivize them to remain loyal to the hotel chain b. the company can conduct a competitive analysis to identify what loyalty programs are offered by its competitors and what their key features and benefits are in order to better position the hotel chains program c. the company can segment its customer base to better understand what types of consumers are more likely to be interested in a loyalty program and what features would most appeal to them

a. the company can survey its existing customers to gather information on what features and rewards they value most and a loyalty program and what would incentivize them to remain loyal to the hotel chain

a clothing retailer is struggling to drive traffic to its website increase online sales how can the company use metrics and control mechanisms to evaluate the effectiveness of its online marketing efforts a. the company can use web analytic tools to track website traffic and user behavior such as time spent on the site bounce rates and conversion rates in order to identify areas for improvement b. the company can survey customers who have made online purchases to gather feedback on their experience and identify pain points that may be contributing to low conversion rates c. the company can conduct A/B testing on different marketing campaigns such as different ad copy or calls-to-action to determine which strategies are most effective in driving traffic and sales

a. the company can use web analytic tools to track website traffic and user behavior such as time spent on the site bounce rates and conversion rates in order to identify areas for improvement

which of the following is a key element of effective project management a. the identification and management of project risks b. the use of a single project management approach across all projects c. the delegation of project management responsibilities to lower level employees d. the focus on minimizing project costs

a. the identification and management of project risks

the company produces two products product A and product B the company has a limited amount of a key raw material that is required to produce both products the company can either produce 10,000 units of product A and 5000 units of product three or produce 7500 units of product A and 7500 units of product B the company has received a special order for 2000 units of product A at a pricing of $20 per unit what is the opportunity cost of producing 2000 units of product a for the special order a. the revenue that could be earned by selling 2000 units of product B b. the variable cost that could be saved by not producing 2000 units of product B c. the fixed cost that could be saved by not producing 2000 units of product B d. none of the above

a. the revenue that could be earned by selling 2000 units of product B

you are the COO of a large corporation that is struggling to keep up with changing industry trends and competition which of the following approaches would be most effective in implementing organizational change a. the use of a participatory approach that involves employees in the change process b. the use of a top down approach that dictates change from upper management c. the focus on maintaining the status quo to avoid risk d. the delegation of change management to lower level employees

a. the use of a participatory approach that involves employees in the change process

you are a CEO of a manufacturing company that is experiencing a significant decline in sales due to internal conflict among employees the conflict has led to decreased productivity poor communication and low morale which of the following strategies would be most effective in resolving the conflict among employees a. the use of conflict resolution techniques and team building exercises b. the use of threats and punishment to force compliance c. the use of coercion to remove employees who are causing conflict d. the use of top down approach to dictate behavior

a. the use of conflict resolution techniques and team building exercises

which of the following is a key characteristic of entrepreneurship a. the willingness to take risks and pursue new opportunities b. the desire to maintain the status quo and avoid change c. the preference for steady and predictable growth d. the focus on maximizing short term profits

a. the willingness to take risks and pursue new opportunities

you are a CEO of a manufacturing company that is experiencing a significant decline in sales due to internal conflict among employees the conflict has led to decreased productivity poor communication and low morale which of the following leadership styles would be most effective in this situation a. transformational leadership b. autocratic leadership c. laissez fair leadership d. transactional leadership

a. transformational leadership

which of the following best describes systems thinking a. understanding how the different parts of the system work together to achieve the systems goals b. focusing on individual parts of the system in isolation c. understanding how to manipulate the parts of a system to achieve a desired outcome d. focusing on the inputs to a system rather than the outputs

a. understanding how the different parts of the system work together to achieve the systems goals

the company produces two products product A and product B the company has a limited amount of a key raw material that is required to produce both products the company can either produce 10,000 units of product A and 5000 units of product three or produce 7500 units of product A and 7500 units of product B the company has received a special order for 2000 units of product A at a pricing of $20 per unit should the company accept the special order for 2000 units of product a a. yes because the revenue from the special order exceeds the variable and fixed costs of producing 2000 units of product a b. no because the revenue from the special order is less than the variable and fixed costs of producing 2000 units of product a c. no because accepting the special order would require the company to produce fewer units of product B which has a higher contribution margin d. none of the above

a. yes because the revenue from the special order exceeds the variable and fixed costs of producing 2000 units of product a

If the average level of nominal income in a nation is $44,000 and the price level index is 175, the average real income would be about: a) $18,857. b) $25,143. c) $44,000. d) $77,000.

b) $25,143.

A firm's net income for the year was $800,000. Average assets totaled $6 million, and average liabilities totaled $1.2 million. Return on equity was: a) 20% b) 16.7% c) 10% d) 13.3%

b) 16.7%

The following are all determinants of demand except: a) Changes in consumer tastes and preferences. b) Changes in consumer price index. c) Changes in the number of buyers or consumers. d) Changes in income.

b) Changes in consumer price index.

Which of the following is(are) an example of a measure of leverage? a) Debt payout ratio. b) Debt/equity ratio. c) Debt yield. d) Preferred dividend coverage ratio.

b) Debt/equity ratio.

Current U.S. Generally Accepted Accounting Principles and auditing standards require the financial statements of an entity for the reporting period to include: a) Projected earnings for the subsequent period. b) Financial position at the end of the period. c) Current fair values of all assets at the end of the period. d) Earnings and gross receipts of cash for the period.

b) Financial position at the end of the period.

Which of the following is an objective of financial reporting by business enterprises? a) Financial reporting should provide assurance that all liabilities of business enterprises will be paid. b) Financial reporting should provide information about the economic resources of an enterprise, the claims to those resources, and changes in those resources and claims to them. c) The primary focus of financial reporting is information about the assets of the entity. d) Financial reporting should show the timing and amount of future cash dividends to potential investors.

b) Financial reporting should provide information about the economic resources of an enterprise, the claims to those resources, and changes in those resources and claims to them.

Which of the following is not a barrier to entry? a) Patents b) Inefficiency c) Economies of scale d) Ownership of essential resources

b) Inefficiency

Which of the following accounting concepts/principles is most significant in the development of a capitalization policy? a) Original Cost. b) Materiality. c) Matching of revenue and expense. d) Consistency.

b) Materiality.

Which of the following accounts is part of working capital? a) Retained Earnings b) Merchandise Inventory c) Common Stock d) Sales

b) Merchandise Inventory

When a purely competitive industry is in long-run equilibrium, which statement is true? a) Average total cost is less than marginal cost. b) Price and average total cost are equal. c) Marginal cost is at its maximum level. d) Marginal revenue is greater than price.

b) Price and average total cost are equal.

Which of the following statements about the Financial Accounting Standards Board is correct? a) The FASB has the authority to fine a noncompliant firm. b) The FASB follows a due process procedure that permits input from interested parties before a standard is issued. c) The FASB is an agency of the Federal government. d) The FASB is controlled by the American Institute of CPA's. e) None of the above statements is correct

b) The FASB follows a due process procedure that permits input from interested parties before a standard is issued.

This historical figure argued it is morally acceptable to charge more for an item than was paid provided the seller had improved the item in some way or was being compensated for risk. a) David Ricardo b) Thomas Aquinas c) Adam Smith d) John Stuart Mill

b) Thomas Aquinas

The declaration of a cash dividend by the directors results in: a) a decrease in cash and a decrease in retained earnings. b) a decrease in retained earnings and an increase in current liabilities. c) a decrease in net income and an increase in current liabilities. d) a decrease in net income and a decrease in cash.

b) a decrease in retained earnings and an increase in current liabilities.

The capital budgeting analytical technique that calculates the rate of return on the investment based on the impact of the investment on the financial statements is known as the: a) net present value. b) accounting rate of return. c) payback period. d) internal rate of return.

b) accounting rate of return.

Noncurrent, intangible assets such as leasehold improvements, patents, and copyrights are all subject to: a) consolidation. b) amortization. c) depreciation. d) depletion.

b) amortization.

In a make or buy decision, which of the following costs would be considered relevant? a) sunk costs. b) avoidable costs. c) unavoidable costs. d) allocated costs.

b) avoidable costs.

The amount by which government expenditures exceed revenues during a particular year is the: a) public debt. b) budget deficit. c) Full employment. d) GDP gap.

b) budget deficit.

When a firm doubles its inputs and finds that its output has more than doubled, this is known as: a) constant returns to scale. b) economies of scale. c) a violation of the law of diminishing returns. d) diseconomies of scale.

b) economies of scale.

An example of a cost likely to have a mixed behavior pattern is: a) raw material cost. b) electricity cost for the manufacturing plant. c) sales force commission. d) depreciation of production equipment.

b) electricity cost for the manufacturing plant.

Cash expenditures a firm makes to pay for resources are called: a) normal profit. b) explicit costs. c) implicit costs. d) opportunity costs.

b) explicit costs.

Which of the following terms do not appear on the contribution margin format income statement? a) operating income. b) gross profit. c) contribution margin. d) variable expenses.

b) gross profit.

The discount rate used to determine the present value of an investment proposal being analyzed is also known as the: a) payback rate. b) hurdle rate. c) present value ratio. d) earnings growth rate.

b) hurdle rate.

One of the most important reasons for having a system of internal control is to: a) ensure no employees have ever been convicted of stealing. b) improve the effectiveness and efficiency of the operations of the organization. c) eliminate any temptations that may be presented to employees that could lead to theft from the company. d) prevent a salesperson from using a company car for personal transportation.

b) improve the effectiveness and efficiency of the operations of the organization.

In the seller's records, the sale of merchandise on account would: a) increase assets and increase paid-in capital. b) increase assets and increase expenses. c) increase assets and decrease revenues. d) increase assets and decrease liabilities.

b) increase assets and increase expenses.

The entry to record depreciation expense: a) decreases a contra asset and decreases net income. b) increases a contra asset and decreases net income. c) decreases an asset and increases a contra asset. d) decreases working capital and decreases net income.

b) increases a contra asset and decreases net income.

The inventory turnover calculation: a) is wrong unless cost of goods sold is used in the numerator. b) is an alternative way of expressing the number of days' sales in inventory. c) requires knowledge of the inventory cost flow assumption being used. d) is wrong unless sales is used in the numerator.

b) is an alternative way of expressing the number of days' sales in inventory.

The return on investment measure of performance: a) is never as important a measure of management effectiveness as the amount of net income. b) is calculated using net income as the amount of return. c) relates dividends paid to the entity's assets. d) is calculated by dividing average assets for a period by the amount of net income for the period.

b) is calculated using net income as the amount of return.

For capital budgeting decisions, the use of present value analysis significantly improves management decision making, however: a) a relevant cost analysis should always be used in close decisions. b) most decisions are significantly influenced by top management's values and experiences. c) the accounting rate of return technique in usually more dependable. d) other quantitative techniques may be even more insightful.

b) most decisions are significantly influenced by top management's values and experiences.

If a purely competitive firm is producing at the MR = MC output level and earning an economic profit, then: a) the selling price for this firm is above the market equilibrium price. b) new firms will enter this market c) some existing firms in this market will leave. d) there must be price fixing by the industry's firms.

b) new firms will enter this market

A potential creditor's judgment about granting credit would be most influenced by the potential customer's: a) most recent acid-test ratio. b) practice with respect to taking cash discounts offered by current suppliers. c) trend of acid-test ratio over the past three years. d) current ratio at the end of the prior fiscal year.

b) practice with respect to taking cash discounts offered by current suppliers.

The total budget variance is caused by two factors: a) time and materials. b) quantity and price. c) direct and indirect relationships. d) fixed and variable cost behavior.

b) quantity and price.

A cash budget would include: a) building depreciation. b) sale of common stock. c) gain on sale of equipment. d) accounts receivable.

b) sale of common stock.

The advantage of a continuous budget is: a) that it saves time for other management activities. b) that the final budget for any quarter should be more accurate. c) that it is less costly to produce. d) that it eliminates the need for annual planning.

b) that the final budget for any quarter should be more accurate.

Recognition of revenue in accrual accounting requires: a) that cash be received. b) that the revenue be realized or realizable, and earned. c) only that the amount of cash to be received from the sale of a product or service be known. d) only that a product be delivered or a service be performed.

b) that the revenue be realized or realizable, and earned.

The net book value of a depreciable asset is: a) the difference between the asset's cost and depreciation expense. b) the difference between the asset's cost and accumulated depreciation. c) the amount for which the asset should be insured. d) the fair value of the asset.

b) the difference between the asset's cost and accumulated depreciation.

The going concern concept refers to a presumption that: a) top management of the entity will not change in the coming year. b) the entity will continue to operate in the foreseeable future. c) the entity will not be involved in a merger within a year. d) the entity will be profitable in the coming year.

b) the entity will continue to operate in the foreseeable future.

When a company splits its common stock 3 for 1: a) retained earnings is decreased by the market value of the shares issued. b) the market value of the company's stock normally falls by two-thirds. c) total paid-in capital increases by a factor of 3. d) the stockholders are assured of receiving larger cash dividends.

b) the market value of the company's stock normally falls by two-thirds.

What is the purpose of conducting a swot analysis? a) to identify the company's internal opportunities and threats b) to identify opportunities and threats in the market environment c) to develop a marketing strategy for a specific product or service d) to measure the effectiveness of the company's marketing efforts

b) to identify opportunities and threats in the market environment

Activity-based costing minimizes the risk of cost distortion when applying overhead by: a) using absorption cost rates. b) using multiple cost driver rates. c) using actual activity cost rates. d) using a single cost driver rate.

b) using multiple cost driver rates.

The part of the variable overhead budget variance due to the difference between actual variable overhead cost and the standard cost allowed for the actual inputs used is called the: a) variable overhead budget variance. b) variable overhead spending variance. c) variable overhead volume variance. d) variable overhead efficiency variance.

b) variable overhead spending variance.

you are the CFO of a manufacturing company that is considering an investment in new production equipment the equipment has a cost of $5 million and is expected to generate annual cash flows of 1.5 million for the next five years the equipment has a salvage value of $500,000 at the end of the fifth year the company's cost of capital is 10% what is the net present value of this investment rounded to the nearest thousand should the company invest in the equipment a. $900,000 yes b. $1,000,000 yes c. $1,000,000 no d. negative $1,000,000 no

b. $1,000,000 yes

A company has a fixed cost of $100,000 and a variable cost of $10 per unit the company sells its products for $30 per unit what is the company's contribution margin per unit a. $10 b. $20 c. $30 d. $40

b. $20

a company is preparing its cash budget for the upcoming quarter the company sales are expected to be $500,000 in January $600,000 in February and $700,000 in March the company's cost of goods sold is 60% of sales the company's beginning cash balance is $50,000 and the company desires to maintain a minimum cash balance of $25,000 what is the company's budgeted cost of goods sold for February a. $350,000 b. $360,000 c. $450,000 d. $540,000

b. $360,000

A monopolized industry should have a Rothschild Index of... a. 0 b. 1 c. 0.5 d. Somewhere between 0 and 1, but not necessarily 0.5.

b. 1

a company has a net income of $500,000 and a total equity of $2 million what is the company's return on equity a. 10% b. 25% c. 50% d. 100%

b. 25%

What would be a good reason to rely on spot exchange for input procurement? a. Spot markets ensure price stability for inputs over time. b. Spot markets avoid the transactions costs associated with drafting contracts. c. Spot exchange allows a company to readily produce its own inputs. d. Spot exchange ensures that you will not run into capacity constraints during periods of peak demand.

b. Spot markets avoid the transactions costs associated with drafting contracts.

what is the difference between a cost plus pricing strategy and a value based pricing strategy a. a value based pricing strategy involves adding a markup to the cost of producing a product or service a cost plus pricing strategy involves setting prices based on the perceived value of the products or services to the customer b. a cost plus pricing strategy involves adding a markup to the cost of producing a product or service a value based pricing strategy involves setting prices based on the perceived value of the product or service to the customer c. a cost plus pricing strategy and a value based pricing strategy have the same meaning and can be used interchangeably d. a cost plus pricing strategy involves setting prices based on the perceived value of the product or service to the customer a value based pricing strategy involves setting prices based on the cost of producing the product or service

b. a cost plus pricing strategy involves adding a markup to the cost of producing a product or service a value based pricing strategy involves setting prices based on the perceived value of the product or service to the customer

Consider the following scenario: Coal is frequently shipped by barge down the Ohio River from the mines in Pennsylvania and West Virginia to power plants dotting the river banks downstream. Suppose Dayton Power and Light (DP&L) builds a dock at its riverside plant to unload the coal delivered by BargeAmerica, an independent shipping company. The type of barge and dock used by BargeAmerica is common among barge companies, but because transportation schedules are irregular and unpredictable, BargeAmerica insists that the dock DP&L employs be reserved only for use by BargeAmerica and may not be used by any of DP&L's other suppliers. The dock that Dayton Power and Light must use to unload the coal delivered by BargeAmerica is an example of... a. physical-asset specificity. b. a dedicated asset. c. vertical integration. d. spot exchange.

b. a dedicated asset.

what is the difference between a direct and indirect marketing channel a. an indirect marketing channel involves selling products or services directly to consumers while in direct marketing channel involves using intermediaries such as wholesalers or retailers to sell products or services b. a direct marketing channel involves selling products or services directly to customers while an indirect marketing channel involves using intermediaries such as wholesalers or retailers to sell products or services c. a direct marketing channel and an indirect marketing channel have the same meaning and can be used interchangeably d. a direct marketing channel involves selling products or services to businesses while an indirect marketing channel involves selling products or services to consumers

b. a direct marketing channel involves selling products or services directly to customers while an indirect marketing channel involves using intermediaries such as wholesalers or retailers to sell products or services

What is the purpose of a marketing audit and how can it be used in strategic marketing planning a. a marketing audit is a financial report of a company's marketing activities it can be used in strategic marketing planning to help a company set financial targets for its marketing efforts b. a marketing audit is a systematic review of a company's marketing activities to identify strengths and weaknesses it can be used in strategic marketing planning to help a company identify areas where it needs to improve its marketing efforts and develop a plan to address those areas c. a marketing audit is a report on the effectiveness of a company's marketing campaigns it can be used in strategic marketing planning to help a company refine its marketing strategies and tactics

b. a marketing audit is a systematic review of a company's marketing activities to identify strengths and weaknesses it can be used in strategic marketing planning to help a company identify areas where it needs to improve its marketing efforts and develop a plan to address those areas

what is the purpose of research hypothesis and how can it be used in market research a. a research hypothesis is a financial projection of the potential return on investment for a research project it can be used in the market research to set financial targets for the project b. a research hypothesis is a statement that predicts the relationship between two or more variables it can be used in market research to guide the research process and to help focus the research on specific questions or issues c. a research hypothesis is a summary of the research findings it can be used in market research to communicate the result of the research to stakeholders d. a research hypothesis is a statement of the research question it can be used in market research to guide the research process and to help focus the research on specific questions or issues

b. a research hypothesis is a statement that predicts the relationship between two or more variables it can be used in market research to guide the research process and to help focus the research on specific questions or issues

which of the following cost allocation methods uses multiple cost drivers to allocate overhead costs to products a. traditional costing b. activity based costing c. target costing d. life cycle costing

b. activity based costing

which of the following costs allocation methods assigns manufacturing overhead costs to products based on the actual conception of overhead resources a. traditional costing b. activity based costing c. target costing d. life cycle costing

b. activity based costing

what is the primary function of investment banks a. providing loans to individuals and small businesses b. advising companies on mergers and acquisitions c. managing the stock and bond portfolios of individuals and institutions d. making loans

b. advising companies on mergers and acquisitions

In an intensively competitive industry, we would expect the Lerner Index for firms to... a. be a negative number. b. be close to 0. c. be close to 1. d. approach infinity.

b. be close to 0.

which of the following is a key element of effective team dynamics a. a focus on individual goals and success b. clear communication and goal alignment c. a rigid hierarchy and control structure d. a lack of diversity among team members

b. clear communication and goal alignment

which of the following financial ratios measures the company's leverage or the amount of debt financing it uses a. acid test ratio b. debt to equity ratio c. return on assets d. price earning ratio

b. debt to equity ratio

a manufacturing company is considering expanding its product line to include a new line of high tech products the company has limited experience with this type of product it is unsure about the market demand for this product you have been hired as a consultant to help the company evaluate this opportunity and make an informed decision what would be the first step for the manufacturing company to evaluate this opportunity a. conduct a SWOT analysis b. determine the potential market size and growth rate c. evaluate the competition and market trends d. identify the costs and resources needed to develop and market the product

b. determine the potential market size and growth rate

which of the following types of analysis focuses on the interesting value of a security a. technical analysis b. fundamental analysis c. quantitative analysis d. market analysis

b. fundamental analysis

which of the following financial statements shows a company's financial performance for a given. a. Balance sheet b. income statement c. statement of cash flows d. statement of changes in equity

b. income statement

which of the following financial statements shows the company's revenues and expenses during a given. a. Balance sheet b. income statement c. statement of cash flows d. statement of changes in equity

b. income statement

which of the following statements is true about absorption costing a. it assigns only variable manufacturing cost to products b. it assigns both variable and fixed manufacturing cost to products c. it treats fixed manufacturing costs. Expenses d. it is not allowed under generally accepted accounting principles

b. it assigns both variable and fixed manufacturing cost to products

which of the following is a key advantage of using competency-based recruiting a. it helps to increase the diversity of the applicant pool b. it helps to ensure that employees are a good fit for the job and the organization c. it helps to reduce the cost of recruiting and selection d. it helps to improve the quality of the applicants

b. it helps to ensure that employees are a good fit for the job and the organization

would you go following variance analysis compares the actual and standard amounts of direct materials and direct labor used in production a. material price and quantity variances b. labor rate and efficiency variances c. overhead spending and efficiency variances d. sale price and volume variances

b. labor rate and efficiency variances

When specialized investments are required to facilitate exchange, the level of the specialized investment actually done is often inefficient compared to the optimum level. The investment actually done is typically... a. more than the optimum level, because firms tend to overestimate the benefits they will get from a unique bilateral exchange relationship. b. less than the optimum level, because firms tend to be unwilling to commit resources to a unique bilateral relationship that may not last long.

b. less than the optimum level, because firms tend to be unwilling to commit resources to a unique bilateral relationship that may not last long.

The greater the need for specialized investment, the more likely it is that the optimal length of a contract for procuring inputs is... a. short. b. long.

b. long.

what is the primary goal of financial management for a publicly traded corporatization a. maximizing profits b. maximizing shareholder value c. maximizing market share d. maximizing employee satisfaction

b. maximizing shareholder value

which of the following is the key factor in employee motivation a. providing high levels of job security b. offering opportunities for personal and professional development c. providing high levels of compensation and benefits d. offering a relaxed and flexible work environment

b. offering opportunities for personal and professional development

a software development company is planning to launch a new product in the market the company has limited resources and is unsure about the best way to market the product to potential customers you have been hired as a consultant to help the company develop a marketing plan that will maximize the products exposure and increase sales which of the following pricing strategies would be most appropriate for the software development company to maximize sales of the new product a. premium pricing b. penetration pricing c. skimming pricing d. discount pricing

b. penetration pricing

which of the following is a key function of capital markets a. providing short term loans to individuals and small businesses b. raising capital for companies and governments c. facilitating international trade d. providing a safe place to store money

b. raising capital for companies and governments

which of the following financial ratios measures the companies efficiency in using its assets to generate revenue a. return on equity b. return on assets c. gross margin ratio d. current ratio

b. return on assets

which of the following budget variances is used to measure the difference between the actual and budgeted levels of activity a. flexible budget variance b. sales volume variance c. direct labor rate variance material price variance

b. sales volume variance

a financial services company is planning to launch a new investment product that targets millennials the company is concerned about the challenges of marketing the new product to this demographic and attracting new customers you've been hired as a consultant to help the company develop marketing plan that will resonate with millennials which of the following communication channels would be most effective for the financial services company to reach millennials a. traditional advertising and marketing channels such as TV and print media b. social media platforms such as Instagram and tik tok c. e-mail marketing and direct mail campaigns d. none of the above

b. social media platforms such as Instagram and tik tok

you are a portfolio manager for a large investment firm that is considering an investment in a startup technology company the company has developed a new software application that is expected to revolutionize the industry however the company is not yet profitable and has a high degree of risk which of the following measures of risk would be the most appropriate to evaluate this investment a. beta b. standard deviation c. expected return d. sharpe ratio

b. standard deviation

which of the following is a key element of effective compensation management a. the use of standardized compensation packages for all employees b. the ability to align compensation with employee performance and organizational goals c. the focus on minimizing compensation costs d. the delegation of compensation management to lower level employees

b. the ability to align compensation with employee performance and organizational goals

which of the following is a key element of effective recruiting and selection a. the focus on hiring for specific technical skills b. the ability to attract and retain top talent c. the use of standardized tests as the primary selection tool d. the delegation of hiring responsibilities to lower level employees

b. the ability to attract and retain top talent

which of the following is a key element of effective systems thinking a. the focus on isolated components and their individual performance b. the ability to understand the independent nature of systems and their components c. the nature of rigid rules and procedures to guide behavior d. the delegation of systems thinking to lower level employees

b. the ability to understand the independent nature of systems and their components

a tech startup that produces a new type of fitness tracker is struggling to gain market share in a highly competitive industry how can the company use buyer behavior concepts to develop a marketing strategy a. the company can conduct surveys and focus groups to better understand consumer motivations for using fitness trackers and how these motivations differ across different consumer groups b. the company can analyze the purchasing behaviors of fitness tracker users to determine which factors are most important in the purchase decision process and how the company's products can better meet these factors c. the company can segment the fitness tracker market based on factors such as age income and lifestyle to develop more targeted marketing campaigns for different consumer groups

b. the company can analyze the purchasing behaviors of fitness tracker users to determine which factors are most important in the purchase decision process and how the company's products can better meet these factors

a fashion retailer is considering expanding its product line to include sustainable and ethical fashion how can the company use market research to inform its marketing strategy a. the company can conduct focus groups to gather feedback from vertical customers on the importance of sustainable and ethical fashion and types of products they would be interested in purchasing b. the company can conduct surveys to gather information on potential customers opinions on sustainable and ethical fashion their purchasing behaviors and their preferred price points c. the company can conduct competitive analysis to identify the strengths and weaknesses of its competitors and the sustainable and ethical fashion market

b. the company can conduct surveys to gather information on potential customers opinions on sustainable and ethical fashion their purchasing behaviors and their preferred price points

a car manufacturer is considering launching a new luxury electric vehicle how can the company use customer surveys to gather information on potential buyers and informants market strategy a. the company can use surveys to gather information on potential customers demographics psychographics and purchasing behaviors b. the company can use surveys to gather information on potential customers opinions of the electric vehicle market their interest in electric vehicles and the features they would like to see in a luxury electric vehicle c. the company can use surveys to gather information on potential customers income levels and willingness to pay for a luxury electric vehicle

b. the company can use surveys to gather information on potential customers opinions of the electric vehicle market their interest in electric vehicles and the features they would like to see in a luxury electric vehicle

Which of the following is a key consideration in supply chain logistics a. the reduction of costs associated with transporting goods and services b. the coordination of activities among multiple organizations in the supply chain c. the optimization of internal organizational processes d. the minimization of inventory holding costs

b. the coordination of activities among multiple organizations in the supply chain

what is the efficient market hypothesis a. the idea that financial markets are completely random and unpredictable b. the idea that financial markets incorporate all publicly available information and their prices c. the idea that financial markets are easily manipulated by large institutions d. the idea that financial markets are characterized by information asymmetry

b. the idea that financial markets incorporate all publicly available information and their prices

According to the text, the HHI for the ready-mix concrete industry is 313. However, ready-mix concrete is expensive to transport far because it's so heavy, so firms rarely compete with other firms outside their local geographic region. Consequently, the HHI of 313 probably makes... a. the ready-mix concrete industry looks more competitive than it really is, and an HHI calculated on the basis of the relevant geographic market would be lower. b. the ready-mix concrete industry looks more competitive than it really is, and an HHI calculated on the basis of the relevant geographic market would be higher. c. the ready-mix concrete industry looks less competitive than it really is, and an HHI calculated on the basis of the relevant geographic market would be lower. d. the ready-mix concrete industry looks less competitive than it really is, and an HHI calculated on the basis of the relevant geographic market would be higher.

b. the ready-mix concrete industry looks more competitive than it really is, and an HHI calculated on the basis of the relevant geographic market would be higher.

which of the following is a key element of effective quality and process management a. the use of employee incentives to increase productivity b. the use of statistical process control to monitor and improve process performance c. the use of technology to automate production processes d. the use of lean manufacturing techniques to minimize waste

b. the use of statistical process control to monitor and improve process performance

a company is considering purchasing a new machine for $50,000 the machine is expected to have a useful life of five years in a salvage value of $5000 the machine is expected to generate annual net cash inflows of $15,000 what is the machines payback. a. Two years b. three years c. four years d. five years

b. three years

what is the primary purpose of options futures and other derivatives a. to generate short term profits b. to manage risk c. to provide long term investment opportunities d. to diversify a portfolio

b. to manage risk

which of the following types of financial instruments is most likely to be traded on the money market a. corporate bonds b. treasury bills c. preferred stock d. common stock

b. treasury bills

which of the following types of securities is the most liquid a. corporate bonds b. treasury bills c. preferred stock d. common stock

b. treasury bills

which of the following methods is used to determine the break even point in units a. contribution margin ratio method b. unit sales method c. absorption costing method d. none of the above

b. unit sales method

which of the following cost behavior describes a cost that varies in total but remains constant per units a. fixed cost b. variable cost c. mixed cost d. step cost

b. variable cost

For 2016, Skresso Co. reported $1.82 of earnings per share of common stock. During 2017, the firm had a 4% common stock dividend. The 2016 earnings per share to be reported in the annual report for 2017 are: a) $1.70 b) $1.90 c) $1.75 d) $1.82

c) $1.75

The following information is from ABC Company's general ledger: Beginning and ending inventories, respectively, for raw materials were $16,000 and $20,000 and for work in process were $40,000 and $44,000. Raw material purchases and direct labor costs incurred were $72,000 each, and manufacturing overhead applied amounted to $40,000. Determine the total cost of goods manufactured during the period. a) $220,000. b) $180,000. c) $176,000. d) $184,000.

c) $176,000.

The following are national income account data for a hypothetical economy in billions of dollars: gross private domestic investment ($320), imports ($35), exports ($22), personal consumption expenditures ($2460), and government purchases ($470). What is GDP in this economy? a) $3,250 billion b) $3,263 billion c) $3,237 billion d) $3,290 billion

c) $3,237 billion

Which of the following statements best describes the process of accounting for depreciation? a) A process for recognizing all of the cost associated with using an asset in a revenue generating activity. b) A process that attempts to recognize loss in economic value over a period of time. c) A process for recognizing the cost of an asset that should be matched against revenue earned as a result of using the asset. d) A process for setting aside cash so funds will be available to replace the asset.

c) A process for recognizing the cost of an asset that should be matched against revenue earned as a result of using the asset.

________ costs between two alternative projects are those that would result from selecting one alternative instead of the other. a) Irrelevant b) Sunk c) Differential d) Allocated

c) Differential

The authoritative financial accounting standards-setting body in the United States is presently the: a) Securities and Exchange Commission (SEC) b) Accounting Principles Board (APB) c) Financial Accounting Standards Board (FASB) d) International Accounting Standards Board (IASB) e) Public Company Accounting Oversights Board (PCAOB)

c) Financial Accounting Standards Board (FASB)

Which of the following captions would be not be reported on a single step income statement? a) Income before taxes b) Net income c) Income from operations d) Net sales

c) Income from operations

Which classification of accounting is most concerned with the use of economic and financial information to plan and control many of the activities of the entity? a) Income tax accounting. b) Auditing-Public accounting. c) Managerial accounting. d) Financial accounting.

c) Managerial accounting.

To which function of management is CVP analysis most applicable? a) Organizing. b) Directing. c) Planning. d) Controlling.

c) Planning.

Computing a borrower's effective interest rate is another application of which of the following concepts? a) Periodic interest concept. b) Current value concept. c) Present value concept. d) None of the above.

c) Present value concept.

Which of the following is not a category of financial statement ratios? a) Financial leverage. b) Profitability. c) Prospectus. d) Liquidity.

c) Prospectus.

Another term for return on investment is: a) Return on equity. b) Return on retained earnings. c) Return on assets. d) Return to sender.

c) Return on assets.

Markets create social welfare by: a) Allowing some consumers to purchase a good for less than they would have been willing to pay. b) Provides some producers to sell a good for more than they would have been willing to sell. c) Some buyers purchase goods for less than they would have been willing and some producers receive a higher amount for a good than they would have been willing to sell. d) Markets do not create social welfare.

c) Some buyers purchase goods for less than they would have been willing and some producers receive a higher amount for a good than they would have been willing to sell.

The balance sheet might also be called: a) Statement of Assets. b) Statement of Changes in Financial Position. c) Statement of Financial Position. d) None of the above.

c) Statement of Financial Position.

Transactions are summarized in: a) The independent auditor's opinion letter. b) The notes for the financial statements. c) The entity's accounts. d) None of the above.

c) The entity's accounts.

Stockholders' equity refers to which of the following? a) A listing of the organization's assets and liabilities. b) Probable future sacrifices of economic benefits. c) The ownership right of the stockholder(s) of the entity. d) All of the above. e) None of the above.

c) The ownership right of the stockholder(s) of the entity.

Most entities satisfy the accounting criteria for recognizing an expense when: a) cash is paid to a supplier. b) a dividend is paid to stockholders. c) a cost is incurred in the revenue generating process. d) a commitment is made to purchase a product or service.

c) a cost is incurred in the revenue generating process.

The term oligopoly indicates: a) a one-firm industry. b) many producers of a differentiated product. c) a few firms producing either a differentiated or a homogeneous product. d) an industry whose four-firm concentration ratio is low.

c) a few firms producing either a differentiated or a homogeneous product.

The relevant range concept refers to: a) a firm's range of sales. b) a firm's range of rates of return. c) a firm's range of activity. d) a firm's range of profitability.

c) a firm's range of activity.

Matching revenues and expenses refers to: a) recording revenues when cash is received. b) recording revenues when a product is sold or a service is rendered. c) accurately reflecting the results of operations for a fiscal period. d) having revenues equal expenses.

c) accurately reflecting the results of operations for a fiscal period.

Management's use of resources can best be evaluated by focusing on measures of: a) liquidity. b) book value. c) activity. d) leverage.

c) activity.

Which of the following lists the components of the master budget in correct chronological order? a) budgeted balance sheet, cash budget, budgeted income statement. b) cash budget, budgeted income statement, budgeted balance sheet. c) budgeted income statement, cash budget, budgeted balance sheet. d) it doesn't matter in which order they are prepared.

c) budgeted income statement, cash budget, budgeted balance sheet.

How can a company use pricing to gain a competitive advantage in the market? a) by offering the lowest prices b) by offering the highest quality product c) by using pricing strategy that is aligned with the company's value proposition and target market d) by offering discounts and promotions to customers

c) by using pricing strategy that is aligned with the company's value proposition and target market

An activity-based costing system involves identifying the activity that causes the incurrence of a cost; this activity is known as a: a) cost object. b) direct cost. c) cost driver. d) cost applier.

c) cost driver.

what is the role of culture in shaping a consumer behavior? a) culture is the primary determinant of consumer behavior b) culture has a minor impact on consumer behavior c) culture influences consumer behavior but is not the only factor d) culture has no impact on consumer behavior

c) culture influences consumer behavior but is not the only factor

Retained earnings represents: a) cash that is available for dividends. b) net income plus gains (or minus losses) on treasury stock transactions. c) cumulative net income of the firm since its beginning that has not been distributed to its stockholders in the form of dividends. d) the total net income of the firm since its beginning.

c) cumulative net income of the firm since its beginning that has not been distributed to its stockholders in the form of dividends.

Craft unions have typically been most effective in raising wage rates by: a) increasing the supply of labor. b) increasing the demand for labor. c) decreasing the supply of labor. d) decreasing the demand for labor.

c) decreasing the supply of labor.

All of the following are examples of negative externalities except: a) noise pollution. b) an industrial factory built in a residential area. c) education. d) coal burning power plants.

c) education.

The impact of changing price levels on amounts reported in financial statements is: a) accomplished by reporting assets at their replacement cost. b) required to be described in the notes to the financial statements. c) encouraged, but not required to be described in the notes to the financial statements. d) reported as a separate item on the balance sheet.

c) encouraged, but not required to be described in the notes to the financial statements.

Management salaries are an example of a ________ cost behavior pattern. a) variable b) mixed c) fixed d) semivariable

c) fixed

Additional paid-in capital is most likely to appear on the balance sheet of a corporation that: a) has issued stock dividends. b) has issued stock at different dates. c) has par value stock. d) has no-par value stock.

c) has par value stock.

The main difference between the short run and the long run is that: a) in the long run, only one variable can be fixed. b) firms earn zero profits in the long run. c) in the short run, one or more inputs are fixed. d) the long run always refers to a time period of one year or longer.

c) in the short run, one or more inputs are fixed.

An advantage of the DuPont model for calculating ROI is that: a) it uses average assets and the straightforward ROI formula does not. b) it is easier to use than the straightforward ROI formula. c) it focuses on asset utilization as well as net income. d) it uses stockholders' equity.

c) it focuses on asset utilization as well as net income.

A magazine publisher has an account called "Unearned Subscription Revenue." The transaction that causes the balance of this account to decrease is: a) magazines are printed for the publisher. b) subscriptions are sold to new subscribers. c) magazines are mailed to subscribers. d) cash is received from new subscribers.

c) magazines are mailed to subscribers.

At the optimal quantity of a public good: a) marginal benefit exceeds marginal cost by the greatest amount. b) total benefit equals total cost. c) marginal benefit equals marginal cost. d) marginal benefit is zero.

c) marginal benefit equals marginal cost.

Another term frequently used to describe stockholders' equity is: a) capital stock. b) gross assets. c) net assets. d) paid-in capital.

c) net assets.

The ZZZ Corporation issued $25 million in new common stock in 2009. It used $18 million of the proceeds to replace obsolete equipment in its factory and $7 million to repay bank loans. As a result, investment: a) of $7 million has occurred. b) of $25 million has occurred. c) of $18 million has occurred. d) has not occurred.

c) of $18 million has occurred.

The budgeting process that most likely creates an attitude supportive of achieving organization goals is: a) zero based approach. b) top-down approach. c) participative approach. d) proportionate increase approach.

c) participative approach.

Current maturities of long-term debt: a) represent cash that has been set aside for debt payments due within a year. b) reflect overdue installments of bonds payable. c) permit a more accurate determination of working capital. d) are classified with long-term debt.

c) permit a more accurate determination of working capital.

If the price index is 130, this means that: a) prices are 130 percent higher than in the base year. b) prices are .13 times higher than in the base year. c) prices are 30 percent higher than in the base year. d) nominal GDP must be inflated to determine the real GDP.

c) prices are 30 percent higher than in the base year.

An organization's system of internal control is designed primarily to: a) ensure that the organization's balance sheet will always balance. b) increase efficiency by letting one employee handle all aspects of a transaction from beginning to end. c) provide an operating framework for all employees as they work to achieve the organization's goals. d) ensure that no employees steal the organization's property.

c) provide an operating framework for all employees as they work to achieve the organization's goals.

An example of a committed cost is: a) charitable contributions. b) manufacturing supplies. c) real estate taxes. d) employee training.

c) real estate taxes.

When a supplier makes a downward adjustment in the amount owed by a creditor, the creditor will: a) increase the amount of the account payable to the supplier, and decrease an asset such as inventory. b) reduce the amount of the account payable to the supplier, and increase cash. c) reduce the amount of the account payable to the supplier, and decrease an asset such as inventory. d) reduce the amount of the account payable to the supplier, and decrease cash.

c) reduce the amount of the account payable to the supplier, and decrease an asset such as inventory.

Return on equity: a) relates dividends and turnover. b) relates dividends and stockholders' equity. c) relates net income and stockholders' equity. d) will be the same as return on investment.

c) relates net income and stockholders' equity.

When comparing entity financial ratios with industry ratios: a) the trend of entity ratios should be compared to the current year's industry ratio. b) it should be assumed that the data result from the consistent application of alternative accounting methods. c) relative values at a point in time may not be significant. d) entity ratios should not be compared with industry ratios.

c) relative values at a point in time may not be significant.

ROI used to evaluate the performance of an investment center manager can sometimes lead to suboptimization. A performance measure designed to avoid the risk of suboptimization is: a) operating income. b) segment income. c) residual income. d) the DuPont model.

c) residual income.

Depreciation, in accounting, is a process that results in: a) accumulating cash for the replacement of the asset. b) depreciable assets being reported in the balance sheet at their fair value. c) spreading the cost of an asset over its useful life to the entity. d) an accurate measurement of the economic usefulness of an asset.

c) spreading the cost of an asset over its useful life to the entity.

One inventory cost flow assumption will result in different cost of goods sold from another inventory cost flow assumption only if: a) price levels do not change during the year. b) a new product is added to inventory during the year. c) the cost of inventory items changes during the year. d) inventory quantities change from the beginning to end of the year.

c) the cost of inventory items changes during the year.

In consolidated financial statements: a) the parent's and subsidiary's financial statements are reported on a separate basis. b) financial statements are reported on an industry-wide basis. c) the parent's and subsidiary's financial statements are reported on a combined basis. d) none of the above.

c) the parent's and subsidiary's financial statements are reported on a combined basis.

Cross elasticity of demand measures how sensitive purchases of a specific product are to changes in: a) the general price level. b) the price of that same product. c) the price of some other product. d) income.

c) the price of some other product.

Accounting can be defined as: a) the process of preparing and auditing an entity's financial statements. b) the process of communicating the financial results of corporate activities to investors and regulatory bodies. c) the process of identifying, measuring, and communicating economic information about an organization for the purpose of making decisions and informed judgments. d) None of the above definitions are correct. e) A, B and C are correct.

c) the process of identifying, measuring, and communicating economic information about an organization for the purpose of making decisions and informed judgments.

The statement of changes in retained earnings for the year shows: a) gains or losses from discontinued operations during the year. b) the effect of a stock split during the year. c) the retained earnings balance at the beginning of the year. d) amounts received from the sale of additional common stock during the year.

c) the retained earnings balance at the beginning of the year.

Price discrimination refers to: a) selling a given product for different prices at two different points in time. b) any price above that which is equal to a minimum average total cost. c) the selling of a given product at different prices that do not reflect cost differences. d) the difference between the prices a purely competitive seller and a purely monopolistic seller would charge.

c) the selling of a given product at different prices that do not reflect cost differences.

The sequence of activities that add value to the organization are: a) the strategic cost initiatives. b) the chain of production events. c) the value chain. d) the value processes.

c) the value chain.

what is the purpose of a research design in market research? a) to collect as much data as possible b) to ensure that the research is ethical c) to ensure that the research is valid and reliable d) to minimize the cost of the research

c) to ensure that the research is valid and reliable

The effect of an adjustment is: a) to record transactions not previously recorded. b) to close the books. c) to increase the accuracy of the financial statements. d) to correct an entry that was not in balance.

c) to increase the accuracy of the financial statements.

When a firm purchases its own shares as treasury stock: a) retained earnings is decreased. b) total stockholders' equity is increased. c) total stockholders' equity is decreased. d) paid-in capital is decreased.

c) total stockholders' equity is decreased.

It is not unusual for a company to use different depreciation methods for book and tax purposes. When this happens, the firm usually: a) is trying to minimize its book income. b) is trying to maximize its taxable income. c) uses an accelerated depreciation method for tax purposes. d) uses an accelerated depreciation method for book purposes.

c) uses an accelerated depreciation method for tax purposes.

The opportunity cost of holding money: a) is zero because money is not an economic resource. b) varies inversely with the interest rate. c) varies directly with the interest rate. d) varies inversely with the level of economic activity.

c) varies directly with the interest rate.

The text asks the question, "Why would a manager ever wish to bear the expense of drafting a contact or have the firm expend resources to integrate vertically and manufacture the inputs itself?" It answers the question, in part, by saying... a. "spot markets do not exist for most kinds of inputs." b. "vertical integration typically circumvents the 'mark-up' problems that tend to arise in procuring inputs, enabling the firm to keep costs considerably lower over the long-term." c. "in the presence of specialized investments, spot exchange does not insulate a buyer from opportunism." d. "managers are not well-trained in these matters, and rarely recognize the complexity of the contracting environment, subjecting them to greater transactions costs."

c. "in the presence of specialized investments, spot exchange does not insulate a buyer from opportunism."

a manufacturing company is considering the purchase of new machinery to increase its production capacity the company has identified a new machine that costs $500,000 and it is expected to increase production by 20% the company expects to sell the additional products for $600,000 resulting in a net profit of $100,000 the company can finance the purchase through a bank loan with an interest rate of 5% over a five year term what is the total interest expense for the bank loan over the five year term a. $100,000 b. $125,000 c. $150,000 d. $175,000

c. $150,000

a startup company has developed a new mobile app that has the potential to disrupt the market the company is seeking funding to launch and market the app the company is considering two financing options equity financing with the venture capital firm that offers a valuation of $10 million and debt financing with a bank at an interest rate of 10% over a five year term what is the total interest expense for the bank loan over the five year term if the company borrows 2 million a. $100,000 b. $200,000 c. $300,000 d. $400,000

c. $300,000

a company is preparing its cash budget for the upcoming quarter the company sales are expected to be $500,000 in January $600,000 in February and $700,000 in March the company's cost of goods sold is 60% of sales the company's beginning cash balance is $50,000 and the company desires to maintain a minimum cash balance of $25,000 how much financing does the company need in February a. $0.00 b. $25,000 c. $35,000 d. $60,000

c. $35,000

a financial services company is considering an investment in a new technology platform for investment management the company has identified A fintech startup that is seeking funding out of valuation of $5 million the company expects to receive a dividend of $200,000 per year over a five year term until it's equity stake at a valuation of $10 million what is the expected return on investment if the financial startup company invests 2 million in the fintech startup a. 20% b. 30% c. 40% d. 50%

c. 40%

a company manufacturers and sells a product with a selling price of $100 per unit the variable cost per unit is $60 and the fixed costs are 200 and $1000 the company has a tax rate of 30% what is the company's break even point in units a. 2500 units b. 4000 units c. 5000 units d. 6667 units

c. 5000 units

Which of the following problems is most associated with "piece rate" compensation? a. Inability to motivate faster production. b. Inability to ensure livable wage standards. c. Inability to ensure quality of production. d. Inability to punish unproductive workers.

c. Inability to ensure quality of production.

Again, Industry A has 4 firms, each with 25% market share, and Industry B has four firm, but one firm has 70% market share, with the others having 10% each. Which of the following is true concerning the Herfindahl-Hirschman Index (HHI)? a. Both industries have the same HHI. b. Industry A has a higher HHI. c. Industry B has a higher HHI.

c. Industry B has a higher HHI.

Dr. Waldron rarely leaves his office, but occasionally engages in an expedition to see what the real world is like. On a recent foray to the Pinnacle Hills Promenade - a shopping mall in Rogers, Arkansas - he observed how many stores sold jeans of all varieties. Over a dozen sellers offered jeans, and more could be found in other stores nearby. Some were inexpensive, others very expensive, and while all the jeans offered had a lot in common, each store's jeans had their own differentiating qualities. Which kind of market structure does it sound like Dr. Waldron has discovered? a. Perfect Competition b. Monopoly c. Monopolistic Competition d. Oligopoly

c. Monopolistic Competition

The text develops an algorithm to help optimize input procurement, asking 2 questions: I. Are there substantial specialized investments relative to contracting costs? II. Is there a complex contracting environment relative to costs of integration? If the answer to both questions is "yes," then the best way to procure inputs would be... a. Spot exchange b. Contract c. Vertical integration

c. Vertical integration

what is the difference between a primary and a secondary reference group a. a primary reference group is a group of people who have a less significant impact on an individual's behavior and attitudes such as coworkers and acquaintances a secondary reference group is a larger group of people who have a significant impact on an individual's behavior and attitudes such as family and close friends b. a primary reference group is a group of people who have no impact on an individual's behavior and attitudes A secondary reference group is a larger group of people who have a significant impact on an individual's behavior and attitudes such as family and close friends c. a primary reference group is a group of people who have a significant impact on an individual's behavior and attitudes such as family and close friends a secondary reference group is a larger group of people who have a less significant impact on an individual's behavior and attitudes such as coworkers and acquaintances d. a primary reference group is a larger group of people who have a significant impact on an individual's behavior and attitudes such as a coworker and acquaintances A secondary reference group is a smaller group

c. a primary reference group is a group of people who have a significant impact on an individual's behavior and attitudes such as family and close friends a secondary reference group is a larger group of people who have a less significant impact on an individual's behavior and attitudes such as coworkers and acquaintances

which of the following is a key element of an effective compensation strategy a. offering the highest salaries and benefits in the industry b. providing A guaranteed level of job security c. aligning employee rewards with business goals and objectives d. minimizing the cost of compensation to the company

c. aligning employee rewards with business goals and objectives

which of the following is a key element of an effective training and development program a. providing training programs only to high performing employees b. focusing on technical skills rather than soft skills c. aligning training programs with the organization's strategic goal and objectives d. offering training programs only to entry level employees

c. aligning training programs with the organization's strategic goal and objectives

which of the following is a key consideration in employment planning a. focusing on the immediate needs of the organization b. reducing the cost of Labor c. anticipating future skills and knowledge requirements d. increasing the number of part time employees

c. anticipating future skills and knowledge requirements

which of the following financial statements reports the company's financial position at a specific point in time a. income statement b. statement of cash flows c. balance sheet d. statement of return to earnings

c. balance sheet

in a transfer pricing decision which of the following factors should be considered a. market price and cost b. negotiation and target profit c. both A and B d. none of the above

c. both A and B

which of the following statements is true about byproduct costing a. by product costs are allocated to the main product b. by product costs are expensed in the period incurred c. byproduct costs are treated as a reduction of the main products cost d. by product costs are allocated to the period in which they are produced

c. byproduct costs are treated as a reduction of the main products cost

which of the following budgets is used to monitor cash flows and outflows during a given period a. Operating budget b. capital budget c. cash budget d. master budget

c. cash budget

which of the following types of costs are considered sunk costs a. future costs that will be incurred b. costs that can be avoided if a particular decision is made c. costs that have already been incurred and cannot be changed d. both A and B

c. costs that have already been incurred and cannot be changed

which of the following is an effective negotiation strategy for maximizing value in a business deal a. distributing values by splitting the difference b. claiming value by asserting bargaining power c. creating value by expanding the pie d. trading value by making concessions

c. creating value by expanding the pie

which of the following financial ratios measures the company's liquidity a. return on assets b. debt to equity ratio c. current ratio d. gross margin ratio

c. current ratio

a small consulting firm has been struggling to increase its revenue despite having a good reputation and experienced consultants you have been hired as a consultant to help the company identify their reasons for its stagnant revenue growth and suggest solutions to overcome the issue which of the following strategies would be most effective for the consulting firm to achieve its revenue growth target a. increase marketing and advertising spend to attract new clients b. offer discounted rates to new clients to encourage them to use their firm's service c. develop new service offerings to meet the needs of existing clients d. focus on improving the efficiency and productivity of the consulting team

c. develop new service offerings to meet the needs of existing clients

a retail chain is experiencing a decline in sales and profitability due to increased competition and changing customer preferences the company has decided to implement a cost cutting program to improve its financial performance you have been hired as a consultant to help the company identify areas where cost cutting can be achieved without compromising the quality of products and services Which of the following strategies would be most effective for the retail chain to cut costs without compromising the quality of products and services a. reduce staff salaries and benefits b. cut back on marketing and advertising expenses c. improve supply chain management and logistics to reduce costs d. decrease the quality of products and services

c. improve supply chain management and logistics to reduce costs

You are a portfolio manager for a wealthy individual who is interested in investing in the stock market the interval is risk adverse and wants to maximize returns while minimizing risk which of the following investment strategies would be appropriate for a risk averse investor a. growth investing b. value investing c. income investing d. momentum investing

c. income investing

which of the following is an advantage of using a just in time inventory system a. higher inventory holding costs b. lower quality control costs c. increased production flexibility d. higher cycle times

c. increased production flexibility

which of the following statements is true about job order costing a. it is used for mass produced goods that are identical or very similar b. it is used for goods that are produced in a continuous flow c. it is used for goods that are customized and made to order d. it is used for goods that are produced in small batches

c. it is used for goods that are customized and made to order

which of the following factors is most likely to affect the level of systematic risk in a portfolio a. the number of securities in the portfolio b. the geographic location of the securities in the portfolio c. macroeconomic conditions d. the type of industries represented in the portfolio

c. macroeconomic conditions

which of the following is a key element of international finance a. maximizing domestic market share b. avoiding global competition c. managing foreign exchange risk d. locating on short term financial gains

c. managing foreign exchange risk

which of the following financial intermediaries provides a mean of pooling small investments and investing in a diversified portfolio of stocks bonds and other securities a. investment banks b. commercial banks c. mutual funds d. hedge funds

c. mutual funds

which of the following methods of capital budgeting is considered the most reliable and accurate for long term investment decisions a. payback period b. Internal rate of return c. net present value d. profitability index

c. net present value

a company has a debt to equity ratio of 1.5 which of the following statements is true a. the company has more liabilities than assets b. the company has more equity than liabilities c. none are true d. the company's total equity is equal to 1.5 times its total liabilities

c. none are true

a financial services company is considering investing in a new fintech startup that has developed a new technology platform for investment management the company is concerned about the risks associated with investment and wants to evaluate the potential return on investment you have been hired as a consultant to help the company make an informed decision which of the following financial ratios would be most appropriate for the financial services company to evaluate the financial health and performance of the fintech startup a. return on investment b. debt to equity ratio c. price to earnings ratio d. gross profit margin

c. price to earnings ratio

which of the following is a responsibility of the Securities and Exchange Commission a. enforcing tax laws b. regulating banking institutions c. regulating stock exchanges d. setting accounting standards

c. regulating stock exchanges

which of the following is a key challenge of organizational change and development a. lack of support from external stakeholders b. lack of resources to support the change initiative c. resistance to change by employees d. inadequate communication by senior management

c. resistance to change by employees

which of the following types of costs are classified as period costs a. direct materials and direct labor b. manufacturing overhead c. selling and administrative expenses d. both A and B

c. selling and administrative expenses

According to the text, when it comes to addressing the principal-agent problem between managers and workers, a. time clocks and spot checks are both very effective. b. time clocks are more effective than spot checks. c. spot checks are more effective than time clocks. d. neither time clocks nor spot checks are very effective.

c. spot checks are more effective than time clocks.

which of the following financial statements shows the change in the company's cash and cash equivalents during a given. a. Balance sheet b. income statement c. statement of cash flows d. statement of changes in equity

c. statement of cash flows

which of the following is a key element of effective planning a. the focus on rigid adherence to plans and schedules b. the use of standardized planning process for all situations c. the ability to anticipate and respond to changes in the business environment d. the delegation of planning to lower level employees

c. the ability to anticipate and respond to changes in the business environment

which of the following is a key element of effective leadership a. the ability to dictate to others what they should do b. the focus on personal success and advancement c. the ability to inspire and motivate others d. the delegation of responsibility to lower level employees

c. the ability to inspire and motivate others

which of the following is a key element of effective organizational change and development a. the focus on maintaining the status quo and avoiding change b. the use of coercive tactics to force change c. the ability to manage resistance to change and facilitate adoption d. the delegation of change management responsibilities to lower level employees

c. the ability to manage resistance to change and facilitate adoption

which of the following is a key element of effective negotiation a. the use of coercive tactics to win concessions b. the ability to control the negotiation process and dictate terms c. the ability to understand the needs and goals of both parties d. the use of ultimatums and threats to force agreements

c. the ability to understand the needs and goals of both parties

which of the following is a key element of effective motivation a. the use or fear and punishment to drive performance b. the focus on individual goals and self interests c. the alignment of employee goals with organizational goals d. the use of external rewards as the primary motivator

c. the alignment of employee goals with organizational goals

Which of the following is a key element of effective organizational structure design a. the focus on maintaining A rigid hierarchy and control structure b. the delegation of structured design to lower level employees c. the alignment of organizational structure with strategy and goals d. the use of a standardized structure for all organizations

c. the alignment of organizational structure with strategy and goals

which of the following is a key element of effective service management a. the minimization of customer contact with service providers b. the use of technology to automate service delivery processes c. the alignment of service delivery with customer needs and expectations d. the focus on maximizing internal efficiency

c. the alignment of service delivery with customer needs and expectations

which of the following is a key element of effective training and development a. the focus on minimizing training costs b. the use of standardized training programs for all employees c. the alignment of training and development programs with organizational goals and employee needs d. the delegation of training and development responsibilities to lower level employees

c. the alignment of training and development programs with organizational goals and employee needs

a furniture retailer is considering opening a new store in a different city how can the company use the supply chain analysis to inform its marketing strategy a. the company can analyze the cost and availability of transportation and logistics in the new city to determine the best method of delivering furniture to customers b. the company can analyze the cost and availability of raw materials in the new city to determine the feasibility of manufacturing furniture locally c. the company can analyze the cost and availability of suppliers and distributors in the new city to determine the best supply chain strategy

c. the company can analyze the cost and availability of suppliers and distributors in the new city to determine the best supply chain strategy

which of the following factors does not affect the cost of debt for a company a. credit ratings of the company b. current market interest rates c. the company's dividend policy d. the companies financial leverage

c. the company's dividend policy

which of the following is a key element of effective quality process management a. the reduction of costs associated with organizational processes and products b. the maximization of shareholder value through cost cutting c. the continuous improvements of organizational processes and products d. the maintenance of the status quo with regard to organizational processes and products

c. the continuous improvements of organizational processes and products

which of the following is a key element of effective supply chain management a. the minimization of transportation costs b. the use of technology to automate supply chain processes c. the development of strong relationships with suppliers and customers d. the maximization of internal efficiency

c. the development of strong relationships with suppliers and customers

you are an operations manager of a manufacturing company that is experiencing decreased productivity and increased waste due to poor quality control processes which of the following strategies would be most effective in improving supply chain logistics a. the minimization of transportation costs b. the use of technology to automate supply chain processes c. the development of strong relationships with suppliers and customers d. the maximization of internal efficiency

c. the development of strong relationships with suppliers and customers

which of the following is a key element of effective control and operations management a. the delegation of control responsibilities to lower level employees b. the use of a single control approach across all levels of the organization c. the measurement and monitoring of key performance indicators d. the focus on reducing costs at all costs

c. the measurement and monitoring of key performance indicators

Again, consider the hypothetical scenario: Coal is frequently shipped by barge down the Ohio River from the mines in Pennsylvania and West Virginia to power plants dotting the river banks downstream. Suppose Dayton Power and Light (DP&L) builds a dock at its riverside plant to unload the coal delivered by BargeAmerica, an independent shipping company. The type of barge and dock used by BargeAmerica is common among barge companies, but because transportation schedules are irregular and unpredictable, BargeAmerica insists that the dock DP&L employs be reserved only for use by BargeAmerica and may not be used by any of DP&L's other suppliers. If Dayton Power & Light signs a long-term contract with BargeAmerica for the delivery of coal, it's probably because... a. there is no spot market for coal. b. the power company wants to vertically integrate into the shipping business. c. the power company wants to avoid giving BargeAmerica an opportunity for a "hold-up." d. long term contracts are less complex than short term contracts.

c. the power company wants to avoid giving BargeAmerica an opportunity for a "hold-up."

which of the following is a key element of effective quality / process management a. the use of employee incentives to increase productivity b. the use of technology to automate production processes c. the use of statistical process control to monitor and improve process performance d. the use of lean manufacturing techniques to minimize waste

c. the use of statistical process control to monitor and improve process performance

you are an operations manager of a manufacturing company that is experiencing decreased productivity and increased waste due to poor quality control processes which of the following approaches would be most effective in implementing effective quality process management a. the use of employee incentives to increase productivity b. the use of technology to automate production processes c. the use of statistical process control to monitor and improve process performance d. the use of lean manufacturing techniques to minimize waste

c. the use of statistical process control to monitor and improve process performance

a software company is developing a new project management tool that it plans to sell to businesses how can the company use segmentation to identify potential target markets for the tool a. the company can divide potential customers into segments based on geographic location and language and then develop different marketing strategies for each segment b. the company can divide potential customers into segments based on their industry and the type of projects they manage and then develop different marketing strategies for each segment c. this company can divide potential customers into segments based on company size and project complexity and then develop different marketing strategies for each segment

c. this company can divide potential customers into segments based on company size and project complexity and then develop different marketing strategies for each segment

which of the following measures of risk considers the profitability of loss a. beta b. standard deviation c. value at risk d. sharpe ratio

c. value at risk

The annual per share dividend requirement of a 6%, $100 par value preferred stock that was issued for $105 is: a) $6.38 b) $10.00 c) $7.50 d) $6.00

d) $6.00

Cassady, Inc. borrowed $25,000 for 3 months at an APR of 10%. The amount of interest paid on this loan was: a) $2.500 b) $600 c) $1,200 d) $625

d) $625

ABC Company's standard direct labor cost per unit includes 3 hours @ $15 per hour. During May ABC Company produced 380 units and incurred total labor cost of $16,200 for 1,200 actual direct labor hours worked. ABC's labor efficiency variance for May is: a) $900 F. b) $0. c) $300 U. d) $900 U.

d) $900 U.

An item that cost $270 is sold for $360. The gross profit ratio for this item is: a) 20% b) 60% c) 33.3% d) 25%

d) 25%

Company A has fixed expenses of $100,000 and variable expenses of $50 per unit. Company B has fixed expenses of $200,000 and variable expenses of $25 per unit. The volume of unit sales necessary to produce exactly the same operating income for Company A and Company B is: a) 5,000. b) 7,500. c) 2,500. d) 4,000.

d) 4,000.

The Consumer Price Index was 115 one year and 120 the next year. The rate of inflation from one year to the next was approximately: a) 2.2 percent. b) 2.9 percent. c) 3.4 percent. d) 4.3 percent.

d) 4.3 percent.

A firm has an ROI of 15%, turnover of 3, and sales of $12 million. The firm's margin is: a) $600,000 b) $1,800,000 c) 30% d) 5%

d) 5%

Which of the following are qualified to express an auditor's opinion about an entity's financial statements? a) A Certified Internal Auditor. b) A Comptroller. c) A Certified Management Accountant. d) A Certified Public Accountant. e) None of the above.

d) A Certified Public Accountant.

For which of the following reconciling items would an adjusting entry be necessary on the company's book? a) An error by the bank. b) Outstanding checks. c) A deposit in transit. d) A bank service charge.

d) A bank service charge.

Sowell indicates that after a natural disaster, price controls will create all of the following except: a) Hoarding of scarce resources such as gasoline, water, and flashlights. b) Shortages of scarce resources. c) The development of black markets. d) Better humanitarian distribution of scarce resources.

d) Better humanitarian distribution of scarce resources.

Which of the following is not a principal category of "Other operating expenses" frequently reported on the income statement? a) Research and development expenses b) General and administrative expenses c) Selling expenses d) Cost of goods sold

d) Cost of goods sold

Which of the following accounts/captions are not ever included in the calculation for Gross Profit? a) Revenues. b) Cost of Goods Sold. c) Net Sales. d) General and Selling Expenses.

d) General and Selling Expenses.

Politicians that indicate they will bring down the costs of medical services are in fact advocating for all of the following except: a) Price ceilings on medical services. b) Lower medical service supply due to lower incentives. c) Shortages of medical services leading to delays. d) Lower costs for nurses and doctors to obtain their education, training and professional certifications.

d) Lower costs for nurses and doctors to obtain their education, training and professional certifications.

Which of the following inventory accounting systems has been made much more feasible as a result of computer systems developments? a) Periodic. b) Physical. c) Just-in-time. d) Perpetual.

d) Perpetual.

________ is a cost management technique in which the firm determines the required cost for a product or service in order to earn a desired profit when the marketplace establishes the product's selling price. a) Differential costing b) Relevant costing c) Product costing d) Target costing

d) Target costing

The major difference between the indirect and the direct method of a statement of cash flows appears in which of the following activities section(s)? a) The investing activities and financing activities sections. b) The operating activities and financing activities sections. c) The investing activities section only. d) The operating activities section only.

d) The operating activities section only.

A favorable materials quantity variance would occur if: a) actual labor hours used was greater than the standard labor hours allowed. b) actual pounds of materials used was greater than the standard pounds allowed. c) more material is purchased than is used. d) actual pounds of materials used were less than the standard pounds allowed.

d) actual pounds of materials used were less than the standard pounds allowed.

How can a company use digital marketing to reach its target audience? a) by creating a website that is optimized for search engines b) by using social media platforms to engaged with customers c) by creating mobile apps for its products or services d) all of the above

d) all of the above

How can a company use technology to improve its marketing effectiveness? a) by implementing social media marketing campaigns b) by using artificial intelligence to analyze customer data c) by developing mobile apps for its products or services d) all of the above

d) all of the above

Indirect costs pertain to costs that: a) are traceable to a cost object. b) are variable costs. c) are commonly incurred. d) are not traceable to a cost object.

d) are not traceable to a cost object.

A standard cost or production standard that is achievable under actual operating conditions is called a(n): a) ideal standard. b) past experience standard. c) average standard. d) attainable standard.

d) attainable standard.

If a firm's payment terms for sales made on account to its customers were 2/10, n30, the number of days' sales in accounts receivable would be expected to be: a) between 25 and 40. b) over 40. c) less than 10. d) between 10 and 25.

d) between 10 and 25.

A performance report for direct labor shows a variance between the budget and actual amounts. This difference is a: a) direct labor spending variance. b) direct labor rate variance. c) direct labor efficiency variance. d) budget variance.

d) budget variance.

Which of the following is true about an unfavorable variance appearing in a performance report? a) budgeted variable costs exceeded actual variable costs. b) actual revenue exceeded budgeted revenue. c) budgeted expense exceeded actual expense. d) budgeted revenue exceeded actual revenue.

d) budgeted revenue exceeded actual revenue.

The current assets of most companies are usually made up of: a) assets that are currently used in the operations of the company. b) a very small proportion (less than 10%) of the total assets of the entity. c) cash, marketable securities, and accounts and notes receivable. d) cash and assets expected to be converted to cash within a year.

d) cash and assets expected to be converted to cash within a year.

Fixed costs classified according to the time frame perspective are known as: a) product cost and period cost. b) constant and inconsistent cost. c) direct cost and indirect cost. d) committed cost and discretionary cost.

d) committed cost and discretionary cost.

The allowance for uncollectible accounts is a(n): a) contra revenue. b) expense. c) asset. d) contra current asset.

d) contra current asset.

Business segment information is included in the notes to financial statements because: a) the amounts shown on the financial statements of most companies are just too large to comprehend. b) net income from various geographic areas can be clearly determined. c) by combining these amounts for each segment, ROI and cash flows for the company as a whole can be determined. d) current and potential investors can make more informed judgments about the company.

d) current and potential investors can make more informed judgments about the company.

Total Fixed Cost (TFC): a) falls as the firm expands output from zero, but eventually rises. b) falls continuously as total output expands. c) varies directly with total output. d) does not change as total output increases or decreases.

d) does not change as total output increases or decreases.

The notes to the financial statements: a) usually disclose the amount of the company's bad debts expense. b) describe management's product development plans for the coming year. c) are not an integral part of the financial statements. d) explain the significant accounting policies of the company.

d) explain the significant accounting policies of the company.

A common size income statement: a) makes comparisons between years more difficult. b) uses the same dollar amount of revenues for each year. c) is useful in estimating the impact of inflation. d) expresses items as a percentage of revenues.

d) expresses items as a percentage of revenues.

If the actual level of activity is different from the budgeted level, a ________ budget is prepared for the actual level of activity: a) zero-based b) continuous c) master d) flexible

d) flexible

Revenue may be recognized: a) from the sale of a company's own common stock. b) in 2016 from the sale of subscriptions of a magazine to be published in 2017. c) if management believes the market value of land held for future development has increased during the year. d) if a company trades inventory at its usual selling price for newspaper advertising.

d) if a company trades inventory at its usual selling price for newspaper advertising.

According to Sowell income distribution is based on: a) race. b) union membership. c) political affiliation. d) income is not distributed, it is earned.

d) income is not distributed, it is earned.

Depreciation expense is not a cash flow item but it will affect the calculation of which cash flow item? a) salvage value. b) working capital. c) initial investment. d) income taxes.

d) income taxes.

A credit entry will: a) increase an expense account. b) decrease paid-in capital. c) increase an asset account. d) increase a liability account.

d) increase a liability account.

A debit entry will: a) increase a liability account. b) increase paid-in capital. c) decrease an asset account. d) increase an expense account.

d) increase an expense account.

In the buyer's records, the purchase of merchandise on account would: a) increase assets and increase expenses. b) have no effect on total assets. c) increase liabilities and increase paid-in capital. d) increase assets and increase liabilities.

d) increase assets and increase liabilities.

A debit entry will: a) always increase the account balance. b) always decrease the account balance. c) increase the balance of a revenue account. d) increase the balance of an expense account.

d) increase the balance of an expense account.

Capital budgeting differs from operational budgeting because: a) capital budgets don't affect cash flow. b) depreciation calculations are required. c) operating expenses are not relevant. d) it considers the time value of money.

d) it considers the time value of money.

Cost accounting is a subset of: a) process cost accounting. b) job order cost accounting. c) financial accounting. d) managerial accounting.

d) managerial accounting.

The accounting concept or principle applied when an allowance is provided for estimated uncollectible accounts receivable is: a) consistency. b) objectivity. c) original cost. d) matching revenue and expense.

d) matching revenue and expense.

The first caption in most income statements in annual reports is: a) gross sales. b) sales, less sales returns and allowances. c) earned revenues. d) net sales.

d) net sales.

The payment of a current liability will: a) increase working capital. b) decrease working capital. c) decrease net income. d) not affect working capital.

d) not affect working capital.

The income statement shows amounts for: a) revenues, expenses, gains, and fair value per share. b) revenues, assets, gains, and losses. c) revenues, expenses, losses, and liabilities. d) revenues, gains, expenses and losses.

d) revenues, gains, expenses and losses.

Paid-in Capital represents: a) earnings retained for use in the business. b) net assets of the entity at the date of the statement. c) fair value of the entity's common stock. d) the amount invested in the entity by the stockholders.

d) the amount invested in the entity by the stockholders.

If the market price of a bond exceeds its face amount: a) the coupon rate is less than the market interest rate. b) the maturity rate has been declining. c) the company's ROI and working capital have been increasing over time. d) the coupon rate is more than the market interest rate.

d) the coupon rate is more than the market interest rate.

When a firm buys land on which there is a building, and the building is torn down so that an appropriate new building can be constructed on the land: a) any of the purchase cost allocated to the old building is reported as a loss. b) any of the purchase cost allocated to the old building is capitalized as part of the cost of the new building. c) the cost assigned to the land excludes the cost of the old building. d) the total cost of the land and old building are capitalized as land cost.

d) the total cost of the land and old building are capitalized as land cost.

Negative externalities arise: a) when the demand curve for a product is located too far to the left. b) only in capitalistic societies. c) when firms pay more than the opportunity cost of resources. d) when firms "use" resources without being compelled to pay for their full costs.

d) when firms "use" resources without being compelled to pay for their full costs.

a financial services company is considering an investment in a new technology platform for investment management the company has identified A fintech startup that is seeking funding out of valuation of $5 million the company expects to receive a dividend of $200,000 per year over a five year term until it's equity stake at a valuation of $10 million what is the expected total return on investment if the financial services company invests 2 million in the fintech startup and sells its equity stake at a valuation of $10 million a. $2.4 million b. $3 million c. $3.6 million d. $4 million

d. $4 million

a manufacturing company is considering the purchase of new machinery to increase its production capacity the company has identified a new machine that costs $500,000 and it is expected to increase production by 20% the company expects to sell the additional products for $600,000 resulting in a net profit of $100,000 the company can finance the purchase through a bank loan with an interest rate of 5% over a five year term what is the expected return on investment for the purchase of the new machinery a. 5% b. 10% c. 20% d. 25%

d. 25%

a startup company has developed a new mobile app that has the potential to disrupt the market the company is seeking funding to launch and market the app the company is considering two financing options equity financing with the venture capital firm that offers a valuation of $10 million and debt financing with a bank at an interest rate of 10% over a five year term what is the percentage of ownership the venture capital firm will have if an invests $5 million at a valuation of $10 million a. 20% b. 25% c. 33% d. 50%

d. 50%

which of the following methods is used to estimate the cost of capital budgeting project a. net present value b. internal rate of return c. payback. d. Both A and B

d. Both A and B

You are a portfolio manager for a wealthy individual who is interested in investing in the stock market the interval is risk adverse and wants to maximize returns while minimizing risk which of the following asset classes would be most appropriate for a well diversified portfolio a. stocks only b. bonds only c. real estate only d. a mix of stocks bonds and other asset classes

d. a mix of stocks bonds and other asset classes

what is the difference between a product and a service a. a product and a service have the same meaning and can be used interchangeably b. a product is tangible and can be held while service is intangible and cannot be held c. a product is something that is consumed by a consumer while a service is something that's provided to a customer d. a product is a physical object that can be purchased while service is an intangible activity or benefit that is provided to a customer

d. a product is a physical object that can be purchased while service is an intangible activity or benefit that is provided to a customer

A healthcare organization is planning to invest in new technology to improve patient care and outcomes the organization has limited funds and is trying to decide whether to finance it investment through debt or equity you have been hired as a consultant to help the organization make an informed decision which of the following factors should the health care organization consider when deciding whether to finance the investment through debt or equity a. the cost of capital b. the level of control and ownership of the organization c. the level of risk associated with the financing option d. all of the above

d. all of the above

a financial services company is considering investing in a new fintech startup that has developed a new technology platform for investment management the company is concerned about the risks associated with investment and wants to evaluate the potential return on investment you have been hired as a consultant to help the company make an informed decision which of the following factors should the financial services company consider when evaluating the investment in the fintech startup a. the level of risk associated with the investment b. the expected return on investment c. the potential for growth and expansion of the fintech startup d. all of the above

d. all of the above

a financial services company is planning to launch a new investment product that targets millennials the company is concerned about the challenges of marketing the new product to this demographic and attracting new customers you've been hired as a consultant to help the company develop marketing plan that will resonate with millennials what element should the marketing plan for the new investment product include a. Emphasis on social responsibility and environmental sustainability b. use of digital marketing and social media channels to reach millennials c. availability of user friendly and convenient online platforms for investment management d. all of the above

d. all of the above

a healthcare organization is experiencing operational inefficiencies and is concerned about the impact on patient care and safety you have been hired as a consultant to help the organization identify areas for improvement and develop an action plan to address the issues which of the following strategies would be most effective for the healthcare organization to improve operational efficiency in patient care a. implement a patient safety program to reduce medical errors and adverse events b. improve the training and development of staff to enhance their skills and knowledge c. automate manual processes and procedures to increase efficiency and reduce errors d. all of the above

d. all of the above

a manufacturing company is considering expanding its product line to include a new line of high tech products the company has limited experience with this type of product it is unsure about the market demand for this product you have been hired as a consultant to help the company evaluate this opportunity and make an informed decision what factors should the company consider when deciding whether to expand its product line to include the new high tech products a. the company's core competencies and capabilities b. the potential profitability of the new product line c. the availability of resources and expertise to develop and market the product d. all of the above

d. all of the above

a manufacturing company is considering the purchase of new machinery to increase its production capacity the company has limited funds and is trying to decide whether to purchase the machinery outright or lease it you've been hired as a consultant to help the company make an informed decision which of the following factors should the manufacturing company consider when deciding whether to purchase or lease the machinery a. the availability of funds and resources to make the purchase b. the expected return on investment from the machinery c. the potential tax benefits of leasing the machinery d. all of the above

d. all of the above

a manufacturing company is experiencing quality issues with one of its products the company has received several customer complaints and is concerned about the impact on its reputation and future sales you have been hired as a consultant to help the company address the quality issues and regain customer confidence what steps should the manufacturer company take to address the quality issues with the product a. conduct a root cause analysis to identify the source of the quality problems b. implement a quality control system to prevent future issues c. communicate openly and transparently with customers about the issue d. all of the above

d. all of the above

a manufacturing company is experiencing quality issues with one of its products the company has received several customer complaints and is concerned about the impact on its reputation and future sales you have been hired as a consultant to help the company address the quality issues and regain customer confidence which of the following strategies would be most effective for the manufacturing company to regain customer confidence and restore its reputation a. offer a discount or refund to customers who have purchased the product b. improve the product quality and reliability c. strengthen customer service and support d. all of the above

d. all of the above

a retail chain is planning to expand its operations to a new geographical region you have been hired as a consultant to help the company develop a market entry strategy that will maximize its chances of success what factors should the retail chain consider when developing a market entry strategy for the new region a. the cultural and economic environment of the new region b. the competitive landscape and market structure in the new region c. the availability of resources and expertise to operate in the new region d. all of the above

d. all of the above

a small consulting firm has been struggling to increase its revenue despite having a good reputation and experienced consultants you have been hired as a consultant to help the company identify their reasons for its stagnant revenue growth and suggest solutions to overcome the issue which of the following steps would be most effective for consulting firm to identify the reasons for stagnant revenue growth a. conduct market research to identify new revenue streams b. evaluate the current pricing strategy and suggest changes if needed c. review the existing client base and identify opportunities to upsell d. all of the above

d. all of the above

a software development company is planning to launch a new product in the market the company has limited resources and is unsure about the best way to market the product to potential customers you have been hired as a consultant to help the company develop a marketing plan that will maximize the products exposure and increase sales which of the following elements should the marketing plan for the new product include a. target market segmentation and positioning b. product pricing and promotion strategies c. sales and distribution channels d. all of the above

d. all of the above

a startup company is planning to enter a new market with a disruptive technology that has the potential to revolution and eyes the industry you've been hired as a consultant to help the company develop a strategy that will enable it to achieve a dominant position in the market what steps should the startup company take to gain a competitive advantage in the market a. conduct market research to identify customer needs and preferences b. develop a unique value proposition that differentiates the product from competitors c. build a strong brand image and reputation d. all of the above

d. all of the above

a startup company is planning to launch a new social media platform that aims to compete with established players in the industry the company is concerned about the challenges of gaining traction in a crowded market and attracting users to its platform you've been hired as a consultant to help the company develop a user acquisition strategy what steps should the startup company take to differentiate its platform from its competitors a. focus on specific niche or target audiences b. offer unique features and functionality that are not available on other platforms c. develop a strong brand image and identity d. all of the above

d. all of the above

a startup company is planning to launch a new social media platform that aims to compete with established players in the industry the company is concerned about the challenges of gaining traction in a crowded market and attracting users to its platform you've been hired as a consultant to help the company develop a user acquisition strategy which of the following strategies would be most effective for the startup company to attract users to its platform a. offer incentives to users who refer their friends to the platform b. invest in targeted advertising and social media marketing c. leverage influencers and brand ambassadors to promote the platform d. all of the above

d. all of the above

a startup company is planning to raise capital to fund its expansion plans the company has limited resources and is considering different financing options to raise the required funds you've been hired as a consultant to help the company decide which financing options to pursue which of the following factors should the company consider when deciding whether to pursue equity or debt financing a. the cost of capital b. the level of control and ownership of the company c. the level of risk associated with the financing option d. all of the above

d. all of the above

a startup company is planning to raise capital to fund its expansion plans the company has limited resources and is considering different financing options to raise the required funds you've been hired as a consultant to help the company decide which financing options to pursue which of the following financing options would be most appropriate for the startup company to raise capital a. equity financing b. debt financing c. crowd funding d. all of the above

d. all of the above

healthcare organization is experiencing operational inefficiencies and is concerned about the impact on patient care and safety you have been hired as a consultant to help the organization identify areas for improvement and develop an action plan to address the issues what steps should the health care organization take to identify areas for improvement a. conduct a process flow analysis to identify bottlenecks and inefficiencies b. gather feedback from staff and patients to identify issues and opportunities for improvement c. review industry best practices and benchmarks to identify areas of improvement d. all of the above

d. all of the above

in a make or buy decision which of the following costs are relevant a. direct materials and labor b. variable manufacturing overhead c. fixed manufacturing overhead d. all of the above

d. all of the above

in a make or buy decision which of the following factors should be considered a. the cost of making the product b. the cost of buying the product c. the capacity of the company's production facilities d. all of the above

d. all of the above

in a sell or process further decision which of the following factors should be considered a. the cost of the original product b. the additional revenue that can be generated by processing the product further c. the additional costs that will be incurred by processing the product further d. all of the above

d. all of the above

where do the following factors affect the price of a call option on a stock a. the stock price b. the exercise price c. the time to expiration d. all of the above

d. all of the above

which of the following factors should be considered when deciding whether to drop a product line a. the products contribution margin b. the products sales volume c. the fixed cost associated with the product line d. all of the above

d. all of the above

which of the following is a limitation of financial statement analysis a. financial statement analysis only provides historical information b. financial statement analysis is subjective c. financial statement analysis does not take into account non financial factors d. all of the above

d. all of the above

which of the following methods is used to allocate the cost of joint products to the individual products a. the physical units method b. the net realizable value method c. the sales value at split off point method d. all of the above

d. all of the above

which of the following methods is used to develop a sales forecast a. time series analysis b. regression analysis c. Market research d. all of the above

d. all of the above

which of the following methods is used to evaluate the performance of a capital budgeting project a. payback period b. Internal rate of return c. net present value d. all of the above

d. all of the above

you are a financial analyst for a large investment bank that is considering the acquisition of a smaller regional bank the regional bank has a strong presence in its local market but has struggled to compete with larger national banks which of the following factors should be considered when evaluating the potential benefits of this acquisition a. increased market share and geographic diversification b. decrease competition and increase market power c. cost saving and increased efficiency d. all of the above

d. all of the above

you are a financial analyst for a large investment bank that is considering the acquisition of a smaller regional bank the regional bank has a strong presence in its local market but has struggled to compete with larger national banks which of the following factors should be considered when evaluating the potential risks of this acquisition a. integration challenges and cultural differences b. regulatory hurdles and legal risks c. financial and operational risks d. all of the above

d. all of the above

you are a financial analyst for a large investment bank that is considering the issuance of a new bond the bond has a face value of $1000 a coupon rate of 5% and a maturity of 10 years the market interest rate is currently 4% which of the following factors should be considered when evaluating the attractiveness of this bond a. the bonds yield to maturity b. the bond credit rating c. the bonds liquidity and trading volume d. all of the above

d. all of the above

you are a financial analyst for a large multinational corporation that is considering a new project in a foreign country the project requires an investment of $10 million and is expected to generate annual cash flow of $3 million for the next 10 years the required rate of return for the project is 15% and the exchange rate is currently one USD = 100 foreign currency units which of the following factors should be considered when evaluating the feasibility of this project a. exchange rate and political risk b. country risk and regulatory risk c. market risk and interest rate risk d. all of the above

d. all of the above

which of the following is a key element of effective inventory management a. maximizing inventory levels to ensure high product availability b. minimizing inventory levels to reduce carrying costs c. placing orders for large quantities of inventory to receive volume discounts d. balancing inventory carrying costs and stock out costs

d. balancing inventory carrying costs and stock out costs

what is the relationship between bond prices and interest rates a. bond prices and interest rates are directly related b. bond prices and interest rates are not related c. the relationship between bond prices and interest rates depends on the creditworthiness of the bond issuer d. bond prices and interest rates are inversely related

d. bond prices and interest rates are inversely related

which of the following methods is used to estimate the cost behavior of a mixed cost a. high low method b. regression analysis c. scatter plot method d. both A and B

d. both A and B

How can a company use competitive intelligence to gain a competitive advantage a. by developing a strategy to copy its competitors products or services b. by undercutting its competitors prices c. by targeting its competitors customers with marketing messages d. by gathering information about its competitors products services and marketing strategies

d. by gathering information about its competitors products services and marketing strategies

which of the following quantitative decision making models is most appropriate for making complex decisions under conditions of uncertainty a. linear programming b. Monte Carlo simulation c. queuing theory d. decision tree analysis

d. decision tree analysis

a retail chain is planning to expand its operations to a new geographical region you have been hired as a consultant to help the company develop a market entry strategy that will maximize its chances of success which of the following market entry strategies would be most appropriate for the retail chain to enter the new region a. exporting b. licensing c. joint venture d. direct investment

d. direct investment

which of the following budgets is used to plan for the purchase of materials and supplies a. sales budget b. production budget c. cost budget d. direct materials budget

d. direct materials budget

According to the text, managers on a fixed salary, even without profit sharing or other company incentives, may be motivated to maximize profits by... a. legal requirements stipulating that managers must maximize profits for owners. b. threats that owners make regarding the safety and welfare of the managers' children. c. altruism and the knowledge that they are doing the right thing. d. external incentives such as their reputation outside the firm and the threat of a takeover.

d. external incentives such as their reputation outside the firm and the threat of a takeover.

which of the following budget variances is used to measure the difference between the actual and budgeted amounts of revenue or expenses a. sales volume variance b. material price variance c. direct labor rate variance d. flexible budget variance

d. flexible budget variance

which of the following statements is true about activity based costing a. it is used only for indirect costs b. it assigns overhead costs to products based on a single cost driver c. it is based on the assumption that all products consume overhead resources in the same way d. it assigns overhead costs to products based on multiple cost drivers

d. it assigns overhead costs to products based on multiple cost drivers

which of the following statements is true about process costing a. it is used for mass produced goods that are identical or very similar b. it is used for goods that are produced in small batches c. it is used for customized goods that are made to order d. it is used for goods that are produced in a continuous flow

d. it is used for goods that are produced in a continuous flow

Of the four market structures on which economists focus, one gets special attention in our text (and substantial attention in our class). This market structure is characterized by "strategic interaction" among a few dominant firms. The text says, "Because large rewards are paid to managers who know how to operate in __________ _________, we will devote two chapters to an analysis of managerial decisions in such markets." To which market structure is the text referring? a. perfectly competitive markets b. monopolistic markets c. monopolistically competitive markets d. oligopolistic markets

d. oligopolistic markets

what is the difference between primary and secondary research a. primary research is research that is conducted by competitors while secondary research is research that is conducted by a company for a specific purpose b. primary research and secondary research have the same meaning and can be used interchangeably c. primary research is research that is conducted by a company for a specific purpose such as consumer surveys or focus groups secondary research is research that is conducted by company to study its own internal operations d. primary research is a research that is conducted by a company for a specific purpose such as consumer surveys or focus groups secondary research is research that has already been conducted by others such as industry reports and academic journals

d. primary research is a research that is conducted by a company for a specific purpose such as consumer surveys or focus groups secondary research is research that has already been conducted by others such as industry reports and academic journals

which of the following financial statements shows the change in the company's retained earnings during a given. a. Balance sheet b. income statement c. statement of cash flows d. statement of changes in equity

d. statement of changes in equity

which of the following is a key element of effective employment planning a. the use of temporary workers is the primary source of Labor b. the focus on minimizing labor costs c. the delegation of employment planning to lower level employees d. the ability to forecast future workforce needs and plan accordingly

d. the ability to forecast future workforce needs and plan accordingly

which of the following is a key element of effective entrepreneurship and small business management a. the focus on maintaining the status quo and avoiding risk b. the use of standardized business practices for all small businesses c. the delegation of entrepreneurship and small business management to lower level employees d. the ability to identify and capitalize on new opportunities

d. the ability to identify and capitalize on new opportunities

which of the following is a key element of effective quantitative decision making models a. the focus on gut instincts and intuition as a primary decision making tool b. the use of standardized decision making models for all situations c. the delegation of decision making to lower level employees d. the ability to use data and analysis to inform decision making

d. the ability to use data and analysis to inform decision making

which of the following is a key element of effective planning a. the development of detailed plans for all possible scenarios b. the use of a single planning approach across all levels of the organization c. the delegation of planning responsibilities to lower level employees d. the alignment of plans with the organization's goal and objectives

d. the alignment of plans with the organization's goal and objectives

which of the following is a key element of effective supply chain logistics a. the minimization of transportation costs b. the use of technology to automate supply chain processes c. the maximization of internal efficiency d. the development of strong relationships with suppliers and customers

d. the development of strong relationships with suppliers and customers

what is the most effective leadership style for empowering employees and promoting initiative a. transactional leadership b. charismatic leadership c. authoritarian leadership d. transformational leadership

d. transformational leadership

which of the following financial instruments has the highest degree of liquidity a. common stock b. corporate bonds c. government bonds d. treasury bills

d. treasury bills

what is the relationship between a company's capital structure and its weighted average cost of capital a. weighted average cost of capital is maximized at the optimal capital structure b. weighted average cost of capital is unaffected by changes in the capital structure c. weighted average cost of capital is only impacted by changes in debt levels d. weighted average cost of capital is minimized at the optimal capital structure

d. weighted average cost of capital is minimized at the optimal capital structure

Over the years, the Securities and Exchange Commission (SEC) has delegated its statutory authority to establish accounting principles for publicly traded companies in the United States to each of the following organizations except the: a) Accounting Principles Board (APB). b) Financial Accounting Standards Board (FASB). c) Committee on Accounting Procedures of the American Institute of Accountants. d) None of the above organizations were ever delegated authority by the SEC to establish accounting principles in the U.S. e) Each of the organizations listed in A, B, and C were at one time or another delegated authority by the SEC to establish accounting principles in the U.S.

e) Each of the organizations listed in A, B, and C were at one time or another delegated authority by the SEC to establish accounting principles in the U.S.

Which of the following is not one of the 5 questions of transaction analysis? a) Does the balance sheet balance? b) Does my analysis make sense? c) Which accounts are affected? d) What's going on? e) Is this an accrual?

e) Is this an accrual?

Budget slack is: a) an allowance for contingencies built into a budget. b) the result of budget estimates submitted that are slightly higher than what c) the costs are really expected to be. d) sometimes called padding or cushion. e) all of the above.

e) all of the above.

Corporate governance includes concerns about: a) the responsibilities of the board of directors. b) business ethics and social responsibility. c) equitable treatment of all stakeholders. d) disclosures and transparency. e) all of the above.

e) all of the above.

The Sarbanes-Oxley Act (SOX) of 2002 does not specifically prohibit an independent auditor from performing the following non-audit function(s) for an audit client: a) "expert" services. b) SOX specifically prohibits an independent auditor from performing all of the non-audit services for an audit client. c) financial information systems design and implementation. d) internal audit outsourcing services. e) tax services.

e) tax services.


Related study sets

Engel v Vitale and Comparison Cases AP Gov

View Set

Chapter 29 Face and Neck Injuries

View Set

CMIS 342 Chapter 5 Quiz and Vocab

View Set

Chapter 4 : Equilibrium - How Supply and Demand Determine Prices

View Set